You are on page 1of 175

A 55-year-old female presents with sudden shortness of breath, tachycardia, and

tachypnea 2 days after surgery. An arterial blood gas reveals a pH of 7.38, PCO2 35, PO2
72, and bicarbonate 23 mEq/L. Lower extremity duplex ultrasonography shows a large clot
in the iliofemoral veins. The diagnosis of pulmonary embolism is made. What would a
chest x-ray most likely show?
1. No abnormality
2. Hampton hump; a cone-shaped area of opacification
3. Westermark sign; a dilated pulmonary artery with distal oligemia
4. Pleural effusion
Answer: 1 - No abnormality
Explanations: The vast majority of patients with acute and hyperacute thromboembolic
disease lack definite radiographic findings. Hampton hump is related to a wedge-shaped
peripheral opacity near the costophrenic angle that points toward the hilum with the base
against the pleura. This represents a pulmonary infarction. The Westermark sign is
dilatation of proximal pulmonary vessels and distal focal oligemia. It is seen in less than 4%
of patients with pulmonary embolism. After 48 hours, at least 30% will show focal
opacities related to developing infarctions or atelectasis. The presence of new pleural
effusion favors infarct over atelectasis.
A 65-year-old female presents to her healthcare provider with the complaint of a cough for
six weeks. She describes the cough as dry and pronounced, especially at night. This cough
is accompanied by hoarseness and a sore throat. On further questioning, she reports a
sensation of burning in her chest and "indigestion" that is worse when she lies down. Her
current medications include lisinopril, metformin, and atorvastatin, all of which she has
been using for several months. She states she had been a smoker for 15 years but quit
recently. She denies any history of allergies, rhinorrhea, wheezing, or difficulty breathing.
What is the most likely cause of her symptoms?
1. Acute pharyngitis
2. Upper airway cough syndrome
3. Gastroesophageal reflux disease (GERD)
4. Medication side effect
Answer: 3 - Gastroesophageal reflux disease (GERD)
Explanations: Gastroesophageal reflux disease (GERD) is a condition of the incompetence
of the lower esophageal sphincter muscle causing the reflux of stomach contents into the
esophagus. Patients with GERD often report symptoms of chest pain, heartburn, sudden
sour taste in the mouth, and a sense of a lump in the throat. These symptoms are often
accompanied by sore throat, laryngitis, chronic cough, and disturbed sleep. Risk factors for
this condition include delayed stomach emptying, pregnancy, obesity, hiatal hernia, and
connective tissue diseases while factors that may aggravate this disorder include smoking,
large late-night meals, fried food, aspirin, etc. Upper airway cough syndrome is
characterized by a chronic cough of greater than or equal to eight weeks duration along
with the sense of something stuck in the throat. Angiotensin-converting enzyme inhibitors
(ACEIs) induced cough is a chronic, dry cough that occurs due to the accumulation of
bradykinin and substance P in the body. Since the patient has been taking the medication
lisinopril for several months with acute symptom onset, GERD is more likely the cause of
cough in this case.
A 67-year-old female presents to her primary care clinician for a routine wellness check.
She is a known smoker with a 39 pack-year history and so of chronic obstructive
pulmonary disease. She states that over the past several weeks she has had increasing
cough secondary to a "cold." The patient states that she also has noticed a lump on the left
side of her abdomen which presented after one of her coughing spells. She denies any
pain associated, but states that sometimes increases in size. On physical exam, the
clinician is unable to appreciate any masses with the patient in the supine or standing
position but thinks that she does have some fullness lateral to her rectus muscle. What is
the next best step to evaluate this possible left-sided fullness?
1. Expectant management and observation
2. MRI of the abdomen
3. CT scan of the abdomen with oral and IV contrast
4. CT scan of the abdomen with IV contrast only
Answer: 3 - CT scan of the abdomen with oral and IV contrast
Explanations: This patient is at risk for a ventral hernia due to the smoking history leading
to chronic obstructive pulmonary disease and her recent coughing spell. Elevated intra-
abdominal pressure can lead to hernias in the anterior abdominal wall, including Spigelian
hernias. CT scan with oral contrast is necessary to evaluate for any possible bowel
contents which may be incarcerated or obstructed secondary to herniation. Physical exam
findings for strangulated Spigelian hernia included significant pain and erythema overlying
the hernia site. The patients may also exhibit signs of bowel obstruction with nausea,
vomiting, and absence of flatus or bowel movements.
A 31-year-old female presents for the third time with a pneumothorax on the left side.
Previously she was treated with a chest tube. She has been having exertional dyspnea for
many months and has been treated for chronic obstructive lung disease. She denies she is
a smoker, does not use drugs, and has no allergies. For the past 3 months, she has
developed constant fatigue, cough, and chest pain. A chest CT scan was done this morning
reveal numerous thin-walled cystic changes in the lung. After the chest tube is inserted in
her, there is discharge (See image). What other comorbid condition is often present in
people with this disorder?
1. Lymphoma
2. Sjogren disease
3. Tuberous sclerosis
4. Leiomyosarcoma
Answer: 3 - Tuberous sclerosis
Explanations: Lymphangioleiomyomatosis (LAM) is a rare disorder that results due to the
proliferation of lymphatics in the lung, axilla, and kidney. It is the abnormal smooth
muscle-like cells that have a neural crest origin and show some features of neoplasia.
There is usually extensive cystic damage to the lung parenchyma, which results in repeated
pneumothoraces, chylous ascites, and chylothorax. LAM is most common in
premenopausal women, indicating that perhaps the female sex hormones play a role in
the pathology. Women who develop LAM are also known to have a high frequency of
tuberous sclerosis. It is estimated that tuberous sclerosis occurs in at least one-third of
LAM patients. Survival for this condition is poor, most women are dead within 10 years
from respiratory failure. Recent data indicate that early diagnosis and better treatment
may extend survival.
A patient is being evaluated for a nonhealing wound in an irradiated field. He had
squamous cell carcinoma of the neck and received 8000 Gy of radiation along with
chemotherapy 12 months ago. Which historical chemotherapy agent would be most
concerning for the development of pulmonary fibrosis during a course of hyperbaric
oxygen therapy?
1. Adriamycin
2. Bleomycin
3. Cis-platinum
4. Doxyrubicin
Answer: 2 - Bleomycin
Explanations: Bleomycin has been implicated in the development of pulmonary fibrosis in
patients exposed to high oxygen tensions. The incidence of pulmonary fibrosis is lower
with lower doses of bleomycin. The exact causal relationship between hyperbaric oxygen
and bleomycin causing pulmonary fibrosis has not been established but it is possible that
bleomycin exposure lowers the lungs' tolerance to pulmonary oxygen toxicity. When
treating a patient with a history of bleomycin exposure with hyperbaric oxygen therapy, it
is prudent to establish baseline pulmonary function and monitor closely during the
treatment course for any signs of toxicity.
A 32-year-old woman is evaluated for exertional dyspnea. Medical history is significant for
systemic sclerosis. Pulmonary examination reveals bilateral fine basal crackles. Pulmonary
function test showed a restrictive pattern with reduced DLCO. CT of the chest showed
increased reticular markings, and ground glass opacification mainly in the lower zones with
no honeycombing changes. What is the most likely diagnosis?
1. Nonspecific interstitial pneumonitis
2. Respiratory bronchiolitis interstitial associated interstitial lung disease
3. Acute interstitial pneumonia
4. Desquamative interstitial pneumonia
Answer: 1 - Nonspecific interstitial pneumonitis
Explanations: Systemic sclerosis-associated interstitial lung diseases are typically those of
fibrotic nonspecific interstitial pneumonia (NSIP) or, less commonly, usual interstitial
pneumonia (UIP). NSIP is a common pattern due to underlying connective tissue disease
which includes systemic sclerosis, polymyositis, dermatomyositis, rheumatoid arthritis,
and Sjogren syndrome. Most common findings on chest CT in NSIP may include increased
reticular markings, traction bronchiectasis, volume loss, and ground glass opacification
mainly in the lower zones. Honeycomb changes are more common with UIP and usually
absent in NSIP.
A 47-year-old female patient with a history of antiphospholipid antibody syndrome who
has been noncompliant with warfarin presents with a deep vein thrombosis and dyspnea.
On examination, her blood pressure is 90/60 mmHg, respiratory rate is 22/min, and her
heart rate is 92/min. Computerized tomography of the chest shows a saddle embolus. She
does not respond to heparin and fluids. An echocardiogram shows right ventricular
hypokinesis. What is the most appropriate next step?
1. Urgent referral for surgical embolectomy
2. Recombinant tissue plasminogen activator
3. Continue administration of fluids and heparin
4. Add lepirudin
Answer: 2 - Recombinant tissue plasminogen activator
Explanations: Anticoagulation alone is an inadequate treatment for this patient. Current
guidelines for the management of unstable patients recommend immediate treatment
with anticoagulation. Fibrinolytic treatment with recombinant tissue plasminogen
activator is indicated unless there are major contraindications. If the patient does not
improve or fibrinolysis is not possible, the best choice is surgical embolectomy. Surgery is
the last resort and never the first consideration. Placing such a patient on bypass is
associated with mortality in excess of 50%. Furthermore, even those who survive are faced
with several complications like acute respiratory distress syndrome (ARDS), bleeding, and
shock. After surgery, the patient will need lifelong treatment with warfarin. Today, tPA is
recommended as the treatment of choice. A dose of 100 mg is given over 2 hours, and
there is a rapid resolution of the elevated pulmonary artery pressure.
A 50-year-old male is brought to the emergency department (ED) after being involved in a
road traffic accident. On arrival, he is intubated immediately for airway protection. His
vitals on arrival include a temperature of 99.2 degrees Fahrenheit, a blood pressure of
100/70 mmHg, a pulse of 110 beats per minute, a respiratory rate of 20 breaths per
minute, and oxygen saturation of 88% on a ventilator. CT abdomen and pelvis shows grade
4 splenic laceration with fluid in the left paracolic gutter. Laboratory investigations reveal a
hemoglobin of 9 g/dl, a white cell count of 12000/mm3, a platelet count of 250000/mm3,
sodium 148 meq/dl, potassium 4 meq/dl, chloride 107 meq/dl, bicarbonate 24 meq/dl,
creatinine 1 mg/dl, and blood urea nitrogen of 20 mg/dl. In the ED, he receives 2 liters
fluid and is taken to the operating room. Open laparotomy is started under general
anesthesia. After 15 minutes into the laparotomy, patient oxygen saturation drops to 70%.
The patient position is double-checked to confirm that endotracheal tube is not kinked.
What is the next best step?
1. Flexible bronchoscopy
2. Rigid bronchoscopy
3. Deflating endotracheal tube balloon and adjust the position of the tube
4. Repeating chest x-ray
Answer: 1 - Flexible bronchoscopy
Explanations: Flexible bronchoscopy is done to rule out any accidental intubation of
aberrant bronchus. Flexible bronchoscopy allows providers to see the relation of the
endotracheal tube cuff to the abnormal bronchus. Based on how far the distance of the
tracheal bronchus from the carina and the endotracheal tube cuff, a decision is made to
use a double-lumen endotracheal tube or use a bronchial blocker. After the appropriate
adjustments are made, flexible bronchoscopy is repeated to confirm the appropriate
positions of the tube. Bilateral breath sounds should be audible on both sides to ensure
appropriate ventilation.
A 35-year-old female recently diagnosed with pulmonary arterial hypertension (PAH)
presents to the clinic for initiating treatment. She reports that she is not able to carry out
any physical activity without being short of breath. She is treatment-naive. Which
medication is going to improve both the World Health Organization (WHO) functional class
and six-minute walk distance (6MWD) for this patient?
1. Continuous intravenous (IV) treprostinil
2. Continuous subcutaneous treprostinil
3. Continuous intravenous (IV) epoprostenol
4. Inhaled treprostinil
Answer: 3 - Continuous intravenous (IV) epoprostenol
Explanations: Continuous IV epoprostenol is the only option that will improve both the
WHO function class and 6MWD in patients with PAH and WHO functional class IV who are
treatment-naive. Continuous IV treprostinil will improve 6MWD but not the functional
class. Epoprostenol has shown to improve PAH symptoms, exercise capacity, and it is the
only treatment that reduces the mortality of patients with idiopathic PAH. Continuous
subcutaneous treprostinil will improve 6MWD but not the functional class in patients with
PAH and WHO functional class IV who are treatment-naive. IV epoprostenol's adverse
effects are flushing, hypotension, dizziness, nausea, and vomiting. Once the patients
develop the side effects, the dose can be decreased until the dose-limiting effects resolve.
There is no renal or hepatic adjustment needed for epoprostenol. Inhaled treprostinil is
the right choice in patients with PAH and WHO functional class IV who are treatment-naive
and unable to tolerate parenteral prostanoid therapy.
An Egyptian male is undergoing anesthesia for surgery of his appendix. Just after induction
of anesthesia, a large worm is seen wriggling in the oral cavity. What complication may
arise if the patient was treated with the appropriate medication at this point?
1. Pharyngeal abscess
2. Bowel obstruction
3. Pneumonitis
4. Ruptured appendix
Answer: 3 - Pneumonitis
Explanations: This patient has ascariasis. Ascaris lumbricoides tend to accumulate in the
lungs and intestines. These parasitic worms grow very large and have an aversion to
anesthetic gasses. Ascaris infection should be treated to prevent complications from
parasite migration, however, during active migration through the lungs medical therapy
can increase the risk of pneumonitis. Albendazole and mebendazole are both effective in
treating adult worms and if given could cause pneumonitis.
A 29-year-old patient with a history of sickle cell disease presents to the emergency
department with difficulty breathing and chest pain. He has a temperature of 101.5F, heart
rate 104, and respiratory rate of 24/minute. The patient is diagnosed with septic shock
secondary to Streptococcus pneumoniae. He received all recommended vaccinations until
age 18. He has not received any vaccinations in the past 10 years. What serotype of S.
pneumoniae is most likely responsible for his condition and what could have prevented
this infection?
1. Streptococcus Pneumoniae serotype 19A, immunization with PPSV23
2. Streptococcus Pneumoniae Serotype 36C, immunization with PSV13
3. Streptococcus Pneumoniae Serotype 13, immunization with PPSV23
4. Streptococcus Pneumoniae Serotype 25A, immunization with PSV13
Answer: 1 - Streptococcus Pneumoniae serotype 19A, immunization with PPSV23
Explanations: An initial dose of PPSV223 should be given at more than 2 years of age after
the completion of the primary PCV13 vaccination series. At a minimum, the PPSV23 should
be given 8 weeks after PCV13. PPSV23 is recommended in all immunocompromised or
asplenic patients. Vaccination is also recommended in patients with high-risk conditions
such as chronic heart disease, cirrhosis, cochlear implants, diabetes mellitus, cerebrospinal
fluid leaks, or chronic lung disease. The most common serotype of Streptococcus
Pneumoniae is serotype 19 and is covered by both PPSV23 and PSV13 vaccinations.
Patients who are given PPSV23 vaccination due to high-risk conditions are recommended
to have a repeat dose of PPSV23 every 5 years.
A 45-year-old man presents for a six-month history of progressive dyspnea and
nonproductive cough. He has smoked one pack per day of cigarettes for 20 years, and he
works as a plumber. Current medications include antacids and acetaminophen as needed.
His oxygen saturation is 92% on room air. Bibasilar crackles without wheezing is noted on
auscultation. The rest of the exam is within normal limits. Pulmonary function tests show
the following: forced vital capacity (FVC) 62% of predicted, forced expiratory volume at 1
second (FEV1) 59% of predicted, total lung capacity 70% of predicted, vital capacity 50% of
predicted, residual volume 70% of predicted, and diffusion lung capacity (DLCO) 49% of
predicted. High-resolution CT (HRCT) shows diffuse ground-glass disease bilaterally with no
enlarged mediastinal or hilar lymphadenopathy. What is the best next step in the
management of this patient?
1. HRCT findings should be sufficient to confirm the diagnosis
2. Bronchoscopy
3. Surgical lung biopsy
4. Initiate antifibrotic therapy
Answer: 3 - Surgical lung biopsy
Explanations: While high-resolution CT findings are helpful in the initial diagnostic workup,
it is often nonspecific when classic radiological findings of idiopathic pulmonary fibrosis
(IPF) are lacking. Classic findings include honeycombing with or without peripheral traction
or bronchiectasis and subpleural basal predominant with heterogeneous distribution.
Bronchoscopy is used to rule out infectious etiology; however, it is not recommended to
confirm the diagnosis due to the inability to obtain adequate lung samples. Transbronchial
biopsy specimens are considered inadequate to diagnose desquamative interstitial
pneumonia (DIP) or to distinguish it clearly from overlapping disease patterns like
respiratory bronchiolitis-associated interstitial lung disease (RB-ILD) in early stages and
nonspecific interstitial pneumonitis (NSIP) in late stages. Surgical lung biopsy is considered
the criterion standard in diagnosing DIP and other types of interstitial lung disease (ILD). In
DIP, open lung biopsy shows infiltration of alveoli with macrophages rich in eosinophilic
cytoplasm and a brown pigment called "smoker's pigment." The initiation of antifibrotics
is only indicated in eligible patients with a confirmed diagnosis of idiopathic pulmonary
fibrosis (IPF).
A 65-year-old female patient who is obese and is a chronic smoker presents seeking help
with smoking cessation. She has tried many times to quit smoking but has been
unsuccessful. Other than eczema, she has no health problems. What is the best way to
counsel the patient for smoking cessation?
1. Start on nicotine replacement products first for at least 3 weeks
2. Set a quit date and stop completely, switching to electronic cigarettes
3. Set up a smoking cessation appointment with a specialized counselor
4. Combine pharmacological treatments with behavioral therapy and other support
Answer: 4 - Combine pharmacological treatments with behavioral therapy and other
support
Explanations: Quitting smoking is not easy, and despite the availability of many products,
none work consistently. Combining behavioral and pharmacological treatments can
increase smoking cessation rates from 8% to14% when compared with minimal behavioral
interventions such as brief advice on quitting. Combination interventions usually include
behavioral components delivered by specialized smoking cessation counselors combined
with nicotine replacement therapy. Combination interventions are made up of at least four
sessions and are more successful with more sessions. Adding behavioral interventions to
pharmacotherapy increases cessation rates from 18% in persons receiving
pharmacotherapy alone to 21% in those using a combination of pharmacotherapy and
behavioral support. There is no solid evidence that electronic cigarettes can help people
quit smoking. Initial studies show that electronic cigarettes contain nicotine and other
harmful chemicals, including carcinogens and lung irritants. The United States Preventive
Services Task Force has found insufficient evidence of the use of electronic cigarettes as a
smoking cessation tool in adults.
A 24-year-old male presents to the office with complaints of numerous red spots on his
skin, lips, and gums. He denies any trauma and denies any medication or illicit drug use.
He says that his brother had the same disorder. He reports recurrent nosebleeds that may
need medical intervention to control. Examination reveals numerous petechiae and
telangiectasia on the skin and the lips. Auscultation reveals a continuous bruit over his
anterior chest. Which complication is he at risk for developing in the future as a result of
his lung findings?
1. Pulmonary hypertension
2. Lung abscess
3. Endocarditis
4. Recurrent pneumothorax
Answer: 1 - Pulmonary hypertension
Explanations: Osler Weber Rendu is also known as hereditary hemorrhagic telangiectasia
and is an autosomal dominant disorder. The disorder usually presents with telangiectasia,
recurrent nosebleeds, and positive family history. Typical findings include red nodules and
starry telangiectasias on the lips. The continuous bruit is due to arteriovenous
malformations (AVMs), and they result in a shunt. Over time this can lead to pulmonary
hypertension. Other complications of pulmonary AVMs include high output failure,
cyanosis, hypoxia, hemoptysis, and cerebral abscess. Once an AVM is diagnosed in the
lung, the patient should be started on prophylactic antibiotics because they are at risk for
developing a cerebral abscess.
A 76-year-old patient with past medical history significant for heart failure with reduced
ejection fraction (HFrEF) with 30% to 35%, chronic obstructive pulmonary disease, and
iron deficiency anemia was admitted for acute dyspnea on exertion with wheezing. CBC
revealed hemoglobin of 6.2 g/dL and hematocrit of 20%. A packed red blood cell
transfusion was initiated. 7 hours later, the patient develops worsening of dyspnea with
oxygen saturation of 72% on room air. Body temperature is reported as 101.2 F. Lung
sounds show rales bilaterally. Neck vein is distended, and the patient developed
hypertension. CBC showed WBC of 10.1 and platelet count of 237/microliter. What is the
most likely diagnosis?
1. Transfusion associated circulatory overload (TACO)
2. Hemolytic transfusion reaction
3. Acute exacerbation of chronic obstructive pulmonary disease
4. Transfusion related acute lung injury (TRALI)
Answer: 1 - Transfusion associated circulatory overload (TACO)
Explanations: TACO and TRALI both can have rales, but TACO has neck distended vein due
to circulatory overload and develops hypertension. TRALI develops hypotension and fever.
Patients with systolic dysfunction are more vulnerable to TACO than TRALI with cardiogenic
pulmonary edema. TRALI likely to show leukopenia and thrombocytopenia.
An 18-year-old male presents to the outpatient clinic for evaluation of a localized skin
abscess. After collecting history and performing a physical exam, the clinician prescribes an
oral antibiotic medication and performs a small incision and drainage of the lesion. On his
way out the door the patient says, "after that experience, I can't wait to smoke my vape."
Which of the following is most accurate regarding the potential health risks this patient
may face in the future?
1. He may see an increase in bronchitic symptoms including coughing and wheezing
2. He may see a decrease in his heart rate and blood pressure
3. He may see a decrease in his risk of coronary artery disease
4. He may see a decrease in his risk of thrombosis
Answer: 1 - He may see an increase in bronchitic symptoms including coughing and
wheezing
Explanations: Studies have shown that adolescents who currently consume e-cigarette
aerosols have a two-fold increase in bronchitic symptoms (cough, wheezing, etc.) when
compared to those who do not consume. Early studies suggest a short-term increase in
vital signs such as blood pressure and heart rate. Traditional cigarettes have shown an
increased risk of acute coronary disease, heart failure, and hypertension with possible side
effects on thrombogenesis. However, the effects of nicotine from e-cigarette delivery
systems are currently limited without any clear conclusions on cardiovascular impacts.
Nicotine alone has been associated with negative cardiovascular outcomes, including
hypertension, development of coronary artery disease, and heart failure.
A 40-year-old female with no significant past medical history presents with nonspecific
respiratory symptoms. A chest x-ray shows a proximal mass. A CT scan shows a polypoid
endobronchial mass measuring 2 cm. Endoscopic resection of the mass is performed. The
microscopic examination reveals a polypoid mass covered by squamous epithelium
without atypical cells or invasion. What is the most likely diagnosis?
1. Squamous cell carcinoma
2. Squamous cell papilloma
3. Glandular papilloma
4. Mixed papilloma
Answer: 2 - Squamous cell papilloma
Explanations: Squamous cell carcinoma is a malignant epithelial tumor characterized by an
infiltrative growth with nests of atypical cells and squamoid pearls. Squamous cell
papilloma is a benign tumor with a fibrovascular core covered by regular squamous
epithelium. Glandular papilloma is a benign tumor with a fibrovascular core covered by
respiratory pseudostratified epithelium. Mixed papilloma is a benign tumor that shares the
features of squamous cell and glandular papillomas.
A 70-year-old female with chronic obstructive pulmonary disease is placed on mechanical
ventilation for respiratory failure. Intubation required rocuronium for paralysis. The
ventilator was placed on assist-control (AC) mode with a rate of 12, a fraction of inspired
oxygen (FIO2) of 1.0, a tidal volume of 500, and positive end-expiratory pressure (PEEP) of
0. Arterial blood gas after intubation demonstrates pH 7.23, PaCO2 75 mmHg, and PO2
350 mmHg, so FIO2 is decreased to 0.70. Half an hour later the patient becomes
hypotensive with a blood pressure of 75/40 mmHg, heart rate 135 beats per minute, and
respiratory rate 26/minute. The trachea is midline, and there are breath sounds in both
lung fields. Bilateral wheezing persists until the next inspiration. The high-pressure alarm
has triggered. What is the preferred initial management?
1. Disconnect the ventilator and manually ventilate the patient
2. Administer intravenous fluid bolus
3. Perform bilateral needle decompression of thorax
4. Start dopamine infusion
Answer: 1 - Disconnect the ventilator and manually ventilate the patient
Explanations: Mechanical ventilation of patients with chronic obstructive pulmonary
disease may develop intrinsic positive end-expiratory pressure also known as auto-positive
end-expiratory pressure (PEEP). Obstructive lung disease requires a longer time for
expiration to avoid the development of auto-PEEP and dynamic hyperinflation. This can be
avoided by having a low inspiratory: expiratory ratio (I:E). As auto-PEEP and dynamic
hyperinflation progress there is an increase in transthoracic pressure. This results in
decreased preload into the right ventricle. As preload decreases you can get resulting
hypotension. The increased alveolar pressure can also result in pneumothorax, but this has
not yet happened in this patient. The ventilator settings will need to be adjusted. There
will need to be a prolonged expiration time. As the patient is overbreathing the ventilator,
she will need to be highly sedated to allow for ventilator synchrony. If needed the
inspiratory flow rate can be increased to help lower the I:E ratio.
A 56-year-old male without a known history of cardiovascular disease presents with
shortness of breath. The patient does not go to the doctor unless he absolutely has to, so
he has not been seen in a medical clinic or hospital for years. He states that he had a
"benign tumor" removed from inside of his chest years ago, he thinks it was from the "lung
surface or something like that," but that he does not remember the name of the tumor.
His provider ordered a lab to help determine if the cause is cardiac or possibly due to a
recurrence of this unknown tumor. Which lab was most likely ordered?
1. C-reactive protein
2. Pro-calcitonin
3. Pro-brain natriuretic peptide
4. Creatinine
Answer: 3 - Pro-brain natriuretic peptide
Explanations: In the evaluation of pleural effusion, determining exudative from
transudative is important to the diagnosis and treatment. Light's criteria can help to
distinguish the type of effusion. Certain labs are also important to consider to determine
the cause of the effusion. C-reactive protein is a non-specific inflammatory marker
elevated in a variety of conditions, both benign and malignant. In the current case of a
patient with no known history of cardiovascular disease and poor clinical follow-up history,
elevated pro-brain natriuretic peptide (BNP) would more likely indicate that this patient
has a pleural effusion due to cardiac reasons than due to a malignant cause. Pro-brain
natriuretic peptide can help to determine cardiac vs. non-cardiac causes of pleural
effusion. The effusion, in this case, would most likely be transudative as well, whereas, in
mesotheliomas, it would more likely be exudative. Pro-calcitonin has been shown to be
helpful in determining the appropriate timing for discontinuing antibiotic use. Creatinine is
helpful in determining kidney function and should be used in conjunction with other
factors such as hydration status, weight, race, age, and muscle mass.
A middle-aged male with long-standing chronic obstructive pulmonary disease (COPD)
presents with dyspnea, fatigue, and copious sputum production to the emergency
department. Twenty-four hours ago, he noted a low-grade fever and a runny nose. Physical
exam reveals respiratory distress with the use of accessory muscles. He has marked
wheezing and no air entry bilaterally. He is immediately treated with oxygen and nebulizer
therapy but fails to improve. The pulmonologist feels that the patient may need a
bronchoscopy for diagnosis and therapeutic reasons. During bronchoscopy, what
percentage of COPD patients with exacerbations have microorganisms in the lower
airways?
1. Less than 3%
2. About 10%
3. About 50%
4. More than 90%
Answer: 3 - About 50%
Explanations: Bronchoscopy is sometimes performed to sample the airways for the
bacteria in patients who are not responding to treatment. Studies suggest that at least
50% of patients have microorganisms in the lower airway during exacerbation of COPD. B.
R. Celli, P. J. Barnes. Exacerbations of chronic obstructive pulmonary disease European
Respiratory Journal Jun 2007, 29 (6) 1224-1238; DOI: 10.1183/09031936.00109906
https://erj.ersjournals.com/content/29/6/1224 At the same time, a significant number of
patients also have colonization of the airways, making it difficult to interpret the data.
However, the one feature is that the bacterial burden increases in the airways during an
exacerbation. New organisms are most likely in patients with symptoms of wet cough and
purulent sputum.
A 36-year-old female presents to the emergency department with complaints of
productive cough, malaise, and fever. She has no known medical history and does not take
any medications. Her blood pressure is 105/92 mmHg, temperature 101.3 F, respiratory
rate 24/min, and heart rate 96/min. Her BMI is 40 kg/m2. This patient's pulse oximetry is
93% on room air. Examination shows jugular venous distention, clear lung fields. A
portable chest x-ray shows low lung volumes but is unremarkable otherwise. Laboratory
results show hemoglobin of 15 g/l, platelets 290000/microL, and leukocytes 5000/microL.
His sodium is 142 mEq/L, potassium 4.3 mEq/L, chloride 99 mEq/L, bicarbonate 35 mEq/L,
BUN 26 mg/dL, creatinine 1.0 mg/dL, and glucose 100 mg/dL. Which of the following is
the best next step in managing this patient?
1. CT chest
2. Arterial blood gas
3. Acetazolamide
4. Ventilation/ perfusion scan
Answer: 2 - Arterial blood gas
Explanations: The patient came to the emergency room with upper respiratory symptoms.
She likely has underlying obesity hypoventilation syndrome. Her serum bicarbonate is
elevated. An ABG on room air should be done on any patient with suspected OHS to look
up for daytime hypercapnia. With her respiratory symptoms, it's possible she may not be
well compensated. An ABG will help determine if this patient needs positive pressure.
A heavy smoker presents with hemoptysis. He has never been to a physician before. He
takes no medications and has no allergies. He has been homeless for the past 5 years. No
other pertinent history is available. A chest x-ray is ordered and the radiologist reports that
the patient has a lesion with an "air-crescent" sign. What is the likely diagnosis?
1. Lung cancer
2. Tuberculosis
3. Aspergillosis
4. Hamartoma
Answer: 3 - Aspergillosis
Explanations: In radiology, the air crescent sign finding on chest x-ray and computed
tomography that is crescenteric and radiolucent is usually due to lung cavitary lesion that
is filled with air and has a round radiopaque mass. Classically, it is due to an aspergilloma,
a form of aspergillosis, that occurs when the fungus Aspergillus grows in a cavity in the
lung. Aspergillosis tends to grow in old lung lesions like scars and tuberculous cavities. The
diagnosis is based on clinical features and serology.
A 65-year-old Vietnam veteran with no medical comorbidities was admitted to intensive care
unit in a tertiary care center following the sudden onset of high-grade fever, intense
headache, cough, and hemoptysis. He was profoundly hypoxic at admission needing
intubation and mechanical ventilation. Inotropes were needed for septic shock. Bloodwork
showed leukocytosis with 13% bands, hemoglobin 12.4 g/dL, mild thrombocytopenia, serum
creatinine, electrolytes, and blood urea were normal. Liver enzymes included aspartate
aminotransferase 70 U/L, alanine aminotransferase 85 U/L, alkaline phosphatase and serum
total bilirubin were normal. CT chest with contrast revealed multilobar infiltrates, most
pronounced in the right middle and lower lobes, associated with a moderate sized effusion. A
week ago, he returned from a week-long stay in Martha's Vineyard at his son's house, where
he also mowed the lawn for one full day and pulverized a rabbit. Blood cultures were drawn,
and he was started on vancomycin, cefepime, and azithromycin. In 48 hours his FiO2 was
increased from 40% to 70%, platelet counts were 42,000/microliter and infiltrates increased
on chest x-ray. What is the most appropriate next step?
1. Add IV doxycycline
2. Change cefepime to meropenem
3. Thoracentesis to rule out empyema
4. Add IV gentamicin
Answer: 4 - Add IV gentamicin
Explanations: Tularemia pneumonia must be included in the differential diagnosis of
severe community-acquired pneumonia because of recent travel to Martha's Vineyard and
using a lawn mower, both of which have been classically associated with previous
outbreaks of pneumonic tularemia. Necrotizing pneumonia occurs following inhalation of
aerosolized Francisella tularensis, either from accidental pulverization of flea infested or
infected rabbits or rodents by a lawn mower or from soil containing excreta of an infected
rodent. Streptomycin or gentamicin are first-line agents for the treatment of tularemia.
Empiric therapy is indicated soon after suspicion arises. Diagnosis is usually made by
serology, due to the high risk of inhalational hazard to the microbiologist by culturing this
organism.
A 17-year-old-male mountain climber and his colleagues are attempting to summit Mount
Everest. After ascending to an altitude of about 5500 meters on the second day, he
complained of headaches, anorexia, nausea, and malaise. On day three of the expedition,
he developed ataxia, impaired cognition, irrational behavior and errors in reading his map.
What is the most likely diagnosis?
1. Acute mountain sickness
2. High-altitude cerebral edema (HACE)
3. High-altitude pulmonary edema (HAPE)
4. Meningitis
Answer: 2 - High-altitude cerebral edema (HACE)
Explanations: Acute mountain sickness (AMS) is a syndrome characterized by headaches,
anorexia, nausea, and malaise. AMS occurs when exposed to altitudes of 4000 meters,
typically for 2 or more days. Headache must be present for a diagnosis of acute mountain
sickness. Treatment includes NSAIDs and supplemental oxygen. Prophylaxis is with
acetazolamide. High-altitude cerebral edema (HACE) is thought to be the end stage form
of acute mountain sickness. The hallmark symptom of HACE is ataxia, but patients may
also experience seizure, altered mental status, or other neurologic symptoms. This patient
has symptoms of acute mountain sickness and ataxia. Hence, he has HACE, not AMS.
Treatment is descent from altitude, corticosteroids, and oxygen. Acetazolamide may also
be started. Symptoms of high-altitude pulmonary edema (HAPE) are breathlessness, chest
pain, headache, fatigue, and dizziness. It occurs at an altitude of about 2000 to 2500
meters, with significant elevation of pulmonary artery pressure leading to capillary
breakdown and subsequent edema.
A 65-year-old male presented with dyspnea and hypoxemic respiratory failure. He has
metastatic stomach cancer with liver, lung, and bone metastases. On presentation, he is
hypotensive, tachypneic, and clearly in distress. He was placed on home hospice 3 weeks
ago with an expected prognosis of less than one month. Chest x-ray showed large right-
sided pleural effusion and moderate left pleural effusion. His son states that he presented
similarly 3 weeks ago and had had fluid removed from his right side. He felt much better
post-procedure. He has a do not resuscitate order placed on the chart. The son wants to
know if something can be done for his father's recurrent pleural effusion. What should he
be told?
1. Thoracentesis now followed by pleural catheter placement
2. Thoracentesis as needed
3. Talc pleurodesis
4. Bedside pleural catheter placement
Answer: 2 - Thoracentesis as needed
Explanations: The choice of therapy depends on the expected survival of the patient. This
patient has malignant pleural effusion, and his life expectancy is less than 1 month.
Therefore, it is less painful if symptomatic relief is provided by repeated thoracentesis. He
may not survive long enough to require another thoracentesis since his last procedure was
3 weeks ago. Thoracentesis should be performed immediately if the acute respiratory
failure is due to pleural effusion. Afterward, an indwelling pleural catheter can be placed if
the patient has recurrent symptomatic pleural effusions, an uncorrectable underlying
cause like malignancy or congestive heart failure, and the patient has expected survival of
more than one month. Talc pleurodesis is only successful in 70% of patients with malignant
pleural effusions; therefore, it is not the first-choice procedure. A bedside pleural catheter
should not be placed because it is tunneled catheter and carries the risk of a tunnel or
pleural infection.
A patient with mild, persistent asthma has been well controlled with low-dose inhaled
corticosteroids. He presents for follow up complaining of some limitation with normal
activities and needing albuterol for symptom relief 3 to 4 days per week over the past
month. Which of the following is the preferred next step?
1. Begin an oral corticosteroid burst
2. Medium-dose inhaled corticosteroid
3. Add a leukotriene receptor antagonist
4. Begin omalizumab
Answer: 2 - Medium-dose inhaled corticosteroid
Explanations: This patient is currently on step 2 of asthma controller therapy, and his level
of control for this scenario is "not well controlled." Therefore, the next step (step 3) is a
medium-dose inhaled corticosteroid. A less preferred option is adding a leukotriene
receptor antagonist, such as montelukast, to the low-dose inhaled corticosteroid. Inhaled
corticosteroids are considered to be superior to leukotriene receptor antagonists when
used as monotherapy. If good control is demonstrated for at least 3 months, a trial of
stepped-down treatment is appropriate. Close follow-up is needed to assess the
effectiveness of the change in treatment.
A 30-year-old G2P1001 presents to the emergency department at 12 weeks gestation with
chest pain and shortness of breath. Her symptoms started two months ago and have been
getting worse. Previous chest x-rays have been noteworthy for cardiomegaly. A chest x-ray
performed in the emergency department reveals an enlarged cardiac shadow but no signs
of pulmonary edema or any acute abnormalities. Pulse oximetry shows a saturation of
97% on room air. An arterial blood gas (ABG) is drawn, revealing a pH of 7.45, paO2 of 105,
paCO2 of 32, and bicarbonate of 20 mEq/L. Complete blood count reveals a hemoglobin of
11.5 mg/dL. The patient's A-a gradient is calculated at 10. Which of the following venous
systems is most likely responsible for these findings?
1. The main coronary venous system
2. The lesser cardiac venous system
3. The greater cardiac venous system
4. The infundibular venous system
Answer: 2 - The lesser cardiac venous system
Explanations: The lesser cardiac venous system is made up of the Thebesian vessels
(veins). The Thebesian veins do not have valves and are present in all 4 cardiac chambers.
The Thebesian veins are embedded in the myocardium in all 4 chambers. The Thebesian
veins serve to both drain the myocardium and contribute to the right to left physiologic
shunting of the body circulation at the level of the heart. The Thebesian veins contribute
to the normal A-a gradient along with the bronchial veins, both draining deoxygenated
blood into the oxygenated blood of the left heart.
An 87-year-old female is admitted to the intensive care unit for severe sepsis secondary to
a urinary tract infection. Despite appropriate fluid resuscitation and antibiotic therapy, she
continues to be lethargic and is intubated due to her inability to maintain her airway. The
ventilator is set to a tidal volume of 360 mL (6 mL/kg), a rate of 12 breaths per min, an
FiO2 of 50%, and a PEEP of 15. Soon after intubation, her blood pressure drops, and she
requires the initiation of inotropic agents. Which of the following maneuvers is most
likely to help improve her blood pressure?
1. Place the patient in a prone position
2. Decrease PEEP to 5
3. Increase FiO2 to 100%
4. Increase the tidal volume to 8 mL/kg
Answer: 2 - Decrease PEEP to 5
Explanations: Patients undergoing mechanical ventilation undergo a change in their
respiratory physiology. In normal respiration, negative intrathoracic pressure caused by
contraction of the diaphragm generates air entry into the lungs. At the same time, the
negative pressure increases venous return to the right atrium. This, in turn, increases the
preload, stroke volume, and cardiac output. In mechanically ventilated patients, this
negative pressure is turned into positive pressure through the ventilator. Positive
intrathoracic pressure effectively "squeezes" the right atrium, decreasing venous return
and diminishing cardiac output. In our case of a septic patient, diminishing cardiac output
by any minute margin can have significant deleterious effects on blood pressure. The
higher the PEEP, the higher the intrathoracic pressure and the higher the effect on cardiac
output. Diminishing PEEP to a more physiologic level can help restore some of the cardiac
output and improve blood pressure in this patient. Of the given options, only decreasing
PEEP would have an effect on blood pressure. None of the other displayed answers would
have any effect on blood pressure.
A 35-year-old female comes into the office with the chief complaint of shortness of breath.
She was diagnosed with asthma four years ago and was treated with albuterol inhalers.
However, she noticed that her dyspnea increased during her recent pregnancy six months
ago. Vital signs are temperature of 37.5 C, blood pressure 120/68 mmHg, heart rate of 69
bpm, and oxygen saturation of 89% on ambient air. Her chest auscultation reveals
diminished breath sounds over the middle lung zone without crackles or wheezing. The
rest of the physical exam is unremarkable. What is the best next step in diagnosis?
1. High resolution computed tomography (HRCT)
2. Oral azithromycin
3. Chest x-ray
4. Complete blood count
Answer: 3 - Chest x-ray
Explanations: High resolution computed tomography (HRCT) is the best radiological
diagnosis for suspected lymphangioleiomyomatosis. However, starting with a chest x-ray is
recommended as an initial test to exclude other diagnoses. The patient does not have
signs of infectious etiology. Although her decreased breath sounds on auscultation could
be due to pneumonia, she will still require a chest x-ray before initiating treatment. Chest
x-ray is a reasonable diagnostic test for the initial diagnosis of lymphangioleiomyomatosis.
Cysts are difficult to be seen on the chest x-ray. However, it can help show pleural effusions
or pneumothorax if present and can help exclude other diagnoses. Complete blood count
would not be the first test to order in this patient. Given her respiratory symptoms and no
indication of systemic disease, CBC is not mandatory at this point.
A 65-year-old male presented to the emergency department with shortness of breath. Initial
arterial blood gas (ABG) showed a pH of 7.31 with a partial pressure of carbon dioxide (PCO2) at
44 mmHg, and partial pressure of oxygen (PO2) at 45 mmHg. A chest x-ray showed bilateral
pulmonary infiltrates. Non-invasive ventilation with a bilevel positive airway pressure mask
(BiPAP) was tried without success. His blood pressure dropped to 80/40 mmHg. He was
intubated immediately. He was treated with antibiotics, vasopressors, and corticosteroids for
septic shock due to bilateral pneumonia. Over the next few days, the patient ‘s clinical status
improved, and he was no longer in need of vasopressors. The patient was placed on a
spontaneous breathing trial. After 40 minutes, his ABG shows a pH 7.39, PCO2 of 38 mmHg, and
PO2 of 63 mmHg on 60% oxygen at 5 cm of water of positive end-expiratory pressure (PEEP). He
is following commands and has mild to moderate airway secretions. Which of the following is
the next best step in the management of this patient?
1. Avoid extubation due to high oxygen requirements and attempt weaning trial the next
morning
2. Extubate patient to non-rebreather mask and consider non-invasive ventilation at night and
during daytime naps
3. Extubate patient to venturi face mask 50% and consider high flow nasal cannula if clinical
status or gas exchange deteriorates
4. Extubate patient to high flow nasal cannula
Answer: 4 - Extubate patient to high flow nasal cannula
Explanations: High flow nasal cannula offers alternative therapy for patients with acute
hypoxic respiratory failure in the immediate post-extubation phase as it has been linked to
positive clinical and physiologic outcomes. High flow nasal cannula allows for high oxygen
supplementation without concerns for airway secretions dryness, which could impair
mucosal cilia clearance and pulmonary toilette. Low flow Oxygen delivery systems such as
face mask do not deliver humidified oxygen, leading to drying of the secretions and
potentially compromising clearance of this patient's secretions during the immediate post-
extubation period. High flow nasal cannula, different from non-invasive ventilation, will not
interfere and facilitate airway secretion clearance. High flow oxygen systems offer the
advantage of providing PEEP to improve oxygenation and alveolar recruitment. This
mechanism of oxygenation improvement is not found with low flow oxygen systems.
A 42-year-old male is climbing the Abruzzi Spur route of K2 on day 7 of the expedition. He
has been complaining of shortness of breath that has progressively worsened over the
course of the last several hours. A portable ultrasound machine is available on the
expedition. What would you expect to see on lung ultrasound?
1. A-line predominant in anterior lung fields
2. Bilateral pleural effusions
3. Absent lung sliding
4. Scattered B-lines throughout all lung fields
Answer: 4 - Scattered B-lines throughout all lung fields
Explanations: Patients with high altitude pulmonary edema can show scattered B-lines on
point-of-care ultrasound imaging. B-lines seen on point-of-care ultrasound can be caused
by increased fluid in the lungs due to pulmonary edema, infection, or heart failure. B-lines
seen on point-of-care ultrasound appear as "comet-tails." A couple B-lines are normal, but
if very large or multiple in a lung field, this is considered abnormal. A line (multiple,
horizontal lines) seen on point-of-care ultrasound are a normal artifact and do not denote
pathology.
A 17-year-old male involved in a knife fight is brought into the emergency department by
emergency medical services with dyspnea and hemoptysis secondary to an anterior neck
and trachea laceration below the thyroid cartilage. His oxygen saturation is 87% and
falling. The patient is unconscious, and despite two attempts at endotracheal intubation, a
definitive airway has not been established. What is the best method of securing this
patient’s airway?
1. Cricothyrotomy
2. Tracheostomy through existing injury
3. Tracheostomy above the level of injury
4. Tracheostomy below the level of injury
Answer: 2 - Tracheostomy through existing injury
Explanations: With a tracheal injury, cricothyrotomy is not indicated due to the cricoid
cartilage being a potential stent maintaining the airway open and the risk of damaging it.
Tracheostomy through an existing injury preserves the most tissue for later repair. Airway
adjuncts above the level of the injury would not be beneficial for obtaining proper
respiration. After tracheal injury repair, a tracheostomy below the level of the injury may
be used to protect the healing area.
A 40-year-old woman presents to your internal medicine clinic with a history of 3 months of a
dry cough. She reports no other medical problems in the past, and she takes no medications.
She migrated from India to the United States 2 years ago. She reports that the cough is dry and
fluctuates, some days it bothers her more than others. She has noticed that the cough is worse
at night. She also reports having an occasional fever of 101 =F over the last 3 weeks. She denies
night sweats, weight loss, and anorexia. She denies symptoms of reflux, sinusitis and post-nasal
drip. On examination, her heart rate is 70 bpm, blood pressure is 110/70 mmHg, respiratory
rate of 14/minute and pulse oximetry of 99% on room air. Her lungs are clear to auscultation.
S1 and S2 heard with no murmurs, abdomen is soft with no tenderness or organomegaly, no
skin rashes are noted. A chest x-ray was done when she started to have this cough 3 months
ago which had shown a small interstitial/nodular opacity in the right lower lobe. At that time, a
Mantoux was done which was negative. Today on repeat chest x-ray, the opacity is no longer
present. A CBC with differential shows the total WBC count to be 13.7 with total eosinophils of
2.1 x 10(9)/L. What would the best test be to make the diagnosis?
1. ANA and ANCA antibody testing
2. Stool for ova and parasites, antibody testing for Strongyloides and Ascaris
3. Interferon-gamma release assay
4. CT of the chest without contrast
Answer: 2 - Stool for ova and parasites, antibody testing for Strongyloides and Ascaris
Explanations: Loeffler syndrome is characterized by transpulmonary passage of helminth
larva through the lungs resulting in fleeting pulmonary opacities and peripheral
eosinophilia. Ascaris larvae are the most common cause worldwide. Loeffler syndrome is
benign, self-limiting without significant morbidity. Anthelminthic drugs are not effective
against the larval stages, so there is no specific treatment for Loeffler syndrome.
A 65-year-old female presents to the emergency department with dyspnea, chills, and
non-radiating right chest pain on inspiration for the past two days. The patient has a
history of well-controlled diabetes mellitus. Her last hemoglobin A1c was 7.1%. The
patient was last seen one month ago for suprapubic pain accompanied by urinary
urgency. She was diagnosed with acute cystitis and prescribed cephalexin. Symptoms
resolved soon after starting this medication. Her home medications are metformin and
a multivitamin she started taking recently. Her heart rate is 88/min, blood pressure
135/87 mmHg, respiratory rate 22/min, temperature 101.2 F, and oxygen saturation is
99% on room air. Physical exam shows bronchial breath sounds in the right lower lobe.
Chest x-ray shows a right lower lobe opacity. Due to recent exposure to beta-lactam
(cephalosporin), she was prescribed levofloxacin. The patient returns seven days later
with minimal symptom improvement. Which of the following is the most likely reason
for her lack of improvement?
1. Decreased fluoroquinolone bioavailability due to chelation effect with cations
2. Medication nonadherence
3. Multidrug-resistant (MDR) organisms
4. Lung abscess
Answer: 1 - Decreased fluoroquinolone bioavailability due to chelation effect with cations
Explanations: Quinolones chelate with tri- and divalent cations such as magnesium
(Mg2+), aluminum (AL2+), calcium (Ca2+), iron (Fe2+), and zinc (Zn2+). This patient was
likely taking her prescribed antibiotic with her multivitamin concurrently, which reduced
the absorption and, therefore, the bioavailability of levofloxacin. Reduced bioavailability
will affect tissue penetration resulting in treatment failure. Patients must take
fluoroquinolone 2 hours before or after multivitamins to avoid this interaction.
Fluoroquinolones directly inhibit bacterial DNA synthesis resulting in bacterial cell death.
Members of this class include moxifloxacin, levofloxacin, ciprofloxacin, ofloxacin,
norfloxacin, and delafloxacin.
A 65-year-old male with history of stage IIIb squamous cell cancer of the lung presents
with increasing dyspnea on exertion and orthopnea ongoing for four weeks with gradual
worsening. On presentation, he is noted to be 90% on ambient air and is tachypneic to a
respiratory rate of 30/minute. Labs are at baseline. A chest x-ray demonstrates large
pleural effusion with contralateral mediastinal shift. He undergoes large-volume
thoracentesis of 3 liters with immediate relief of dyspnea. The next morning, he develops
respiratory distress with new hypoxia requiring supplemental oxygen. What is the most
likely cause of the patient's newly developed respiratory distress?
1. Pneumothorax ex vacuo
2. Hemothorax
3. Reexpansion pulmonary edema
4. Pulmonary embolism
Answer: 3 - Reexpansion pulmonary edema
Explanations: Reexpansion pulmonary edema occurs after the removal of fluid or air from
the pleural space quickly over a short period of time. The mechanism of edema is
believed to be increased capillary permeability. Risk factors for this condition include
young age, a long duration of lung collapse, and rapid reexpansion. Treatment is largely
supportive. Thoracentesis operators should not perform large-volume thoracentesis. No
more than 1.5 liters of fluid should be removed in any instance.
A 58-year-old male is referred for dyspnea on exertion and cough of 3 weeks duration. His
primary care provider had diagnosed him with pneumonia and was prescribed two courses
of antibiotics without significant improvement in symptoms. Vitals are normal except low
oxygen saturation. Lung auscultation reveals intermittent crackles. High-resolution CT of
the chest shows bilateral infiltrates predominantly in the lower lobes along with small
pleural effusions. Which is the most appropriate next test that will enable to make a
diagnosis?
1. Bronchoscopy with bronchoalveolar lavage
2. Serology for IgG antibodies to common allergens
3. Positron emission tomography with fluorodeoxyglucose
4. Angiotensin-converting enzyme level
Answer: 1 - Bronchoscopy with bronchoalveolar lavage
Explanations: Cryptogenic organizing pneumonia (COP) is the most likely diagnosis in this
patient given the persistent pulmonary opacities despite antibiotic treatment. Mixed
cellularity on bronchoalveolar lavage (BAL) with elevated lymphocytes and low CD4:CD8
ratio supports a diagnosis of COP. BAL also helps rule out atypical infections which could
present similarly. A thorough history should be taken to exclude other diagnoses and
exclude secondary organizing pneumonia.
A 66-year-old female patient with a history of obstructive sleep apnea, hypertension,
diabetes mellitus, class 3 obesity, and asthma presents with five days of worsening dyspnea
and associated wheezing and cough. Her vital signs on presentation are blood pressure
168/70 mmHg, oxygen saturation 85% on room air, respiratory rate 32 breaths per minute,
and pulse 110 beats per minute. On exam, she has diffuse expiratory wheezing. A chest x-
ray does not show any significant infiltrates. Her labs are grossly normal, except her arterial
blood gas (ABG) shows a pH of 7.2, pCO2 56 mmHg, pO2 55 mmHg, and bicarbonate of 21
mmol/L. Nebulization and corticosteroids are started. She is placed on BiPAP and
transferred to a step-down unit. Repeat ABG shows a pH of 7.1 and pCO2 of 70 mmHg. She
is prepared to be intubated and moved to the ICU. Rapid sequence intubation is attempted
but fails after four attempts. The patient develops bradycardia then cardiac arrest. Which of
the following could have been the cause of intubation failure?
1. Placement of two fingers in the superior laryngeal notch
2. Placement of three fingers on the floor of the mandible between the mental angle and
the neck near the hyoid bone
3. Placement of two fingers between the incisors
4. Placement of three fingers between the incisors
Answer: 3 - Placement of two fingers between the incisors
Explanations: Measurement of three fingers between the upper and lower teeth of the open
mouth of a patient indicates the ease of access to the airway through the oral opening. A typical
patient can open their mouth sufficiently to permit the placement of three of their fingers
between the incisors. Adequate mouth opening facilitates both insertions of the laryngoscope
and obtaining a direct view of the glottis. Measurement of three fingers from the anterior tip of
the mandible to the anterior neck provides an estimate of the volume of the submandibular
space. A typical patient can place three fingers on the floor of the mandible between the mental
angle and the neck near the hyoid bone. Normally this distance should measure close to 7 cm. If
this distance is less than three finger-widths, the laryngeal axis will be at a more acute angle with
the pharyngeal axis, indicating that alignment of the oral opening to the pharyngeal opening will
be difficult. It also indicates that there will be less space to displace the tongue within the throat.
The rule has limitations as the distance can vary according to height and ethnicity. For this
reason, an alternative in the form of a ratio of height to thyromental distance (RHTMD) has been
suggested. Measurement of two fingers between the floor of the mandible to the thyroid notch
on the anterior neck identifies the location of the larynx relative to the base of the tongue. A
typical patient can place two fingers in the superior laryngeal notch. If the larynx is too high in
the neck, measuring less than two fingers, direct laryngoscopy will be difficult and potentially
impossible; this is because the angle between the base of the tongue to the larynx is too acute to
be negotiated for direct visualization of the larynx easily.
A 55-year-old female presents with an unremitting cough in addition to polyuria,
polydipsia, and menstrual irregularities for the past few months. A chest x-ray shows
perihilar lymphadenopathy. Biopsy of a lymph node reveals non-caseating granulomas.
Which part of the brain most likely is affected that is causing her endocrinologic
problems?
1. Cerebellum
2. Hypothalamus
3. Corpus callosum
4. Cortical lesions
Answer: 2 - Hypothalamus
Explanations: Neurosarcoidosis in rare cases may cause neuroendocrine dysfunction
involving the hypothalamus. Symptoms of neurosarcoidosis may present as neurogenic
diabetes insipidus. The spinal cord can be involved causing a cauda equina clinical
presentation. Endocrinology should be consulted in cases where the hypothalamus is
affected.
A 65-year-old nursing home patient presents with a 10-day history of a productive cough,
shortness of breath, and fever. She has a history of alcohol use disorder and a penicillin
allergy. Chest x-ray shows an irregularly shaped cavity with an air-fluid level in the right
lower lobe. What are the most appropriate antibiotics?
1. Levofloxacin and ampicillin
2. Ceftriaxone and metronidazole
3. Clindamycin and erythromycin
4. Vancomycin, clindamycin, and ceftriaxone
Answer: 4 - Vancomycin, clindamycin, and ceftriaxone
Explanations: The location of the abscess makes anaerobic bacteria most likely.
Clindamycin would provide coverage for anaerobic bacteria. Clindamycin, as empiric
treatment for a lung abscess, is no longer recommended given the risk of Clostridioides
difficile infection but remains an alternative for penicillin-allergic patients. Methicillin-
resistant Staphylococcus aureus is covered with vancomycin and linezolid. The patient
would most likely be hospitalized and started on at 2 to 3 antibiotics. Treatment should be
for 4 to 6 weeks. Chest x-ray changes are gradual, but one will see a decrease in the
abscess cavity.
A SCUBA diver is brought to the emergency department after a dive. He states he is still
training and did his first deep dive today. He states that his instructor told him he may be
ascending a bit fast. He complains of facial swelling, and eye redness that he noticed just
after the dive and complained of his mask feeling too tight. He also complains of a mild
headache and tingling in his arms which he attributes to the cold water and tight mask.
Mild bruising is noted on the face and periorbital areas. Eye movements are normal, and
no double vision is present. Which of the following is the next best step in the
management of this patient?
1. Reassure patient that the bruising and redness in his eyes will improve in a couple of
weeks
2. Order an MRI of the orbits just to be sure there is no orbital hemorrhage
3. Consult ophthalmology to perform a dilated fundoscopic examination
4. Place the patient on oxygen, perform a thorough neurologic exam and contact a
hyperbaric treatment center
Answer: 4 - Place the patient on oxygen, perform a thorough neurologic exam and contact
a hyperbaric treatment center
Explanations: The patient is displaying symptoms concerning for decompression illness. It
is important for the examine to realize that while the patient's injuries from a mask
squeeze are mild, other dive related injuries may occur concurrently. The patient has
evidence suggestive of a decompression illness injury involving the CNS. This would
require supplemental oxygen, a thorough neurologic exam, as well as contacting a
hyperbaric treatment center for further treatment and possible transfer to a facility with a
decompression chamber. This patient's mask squeeze is a mild injury and does not require
a fundoscopic exam.
A 56-year-old female is referred to the clinic from her gynecologist where she had gone to
get cervical cancer screening. During the office visit there, she fell asleep mid-
conversation. Given her body habitus, the gynecologist was concerned for obstructive
sleep apnea and made the referral. Blood work done at the OBGYN visit is significant for
elevated serum bicarbonate at 32 mEq/L. She denies smoking. There is some concern for
obstructive sleep apnea (OSA) and obesity hypoventilation syndrome (OHS). Which of the
following is most accurate about OHS?
1. An alternative neuromuscular, mechanical or metabolic explanation for hypoventilation
can be coexistent
2. Presence of awake alveolar hypoventilation characterized by daytime hypercapnia
(arterial PCO2 greater than 45 mmHg)
3. A consequence of diminished ventilatory drive and capacity related to individual patient
being overweight (BMI greater than 25)
4. Presence of awake hypoxia characterized by arterial PO2 less than 65 mmHg
Answer: 2 - Presence of awake alveolar hypoventilation characterized by daytime
hypercapnia (arterial PCO2 greater than 45 mmHg)
Explanations: OHS is defined as the presence of awake alveolar hypoventilation
characterized by daytime hypercapnia (arterial PCO2 greater than 45 mm Hg [5.9 kPa])
that is thought to be a consequence of diminished ventilatory drive and capacity related to
obesity (BMI greater than 30) in the absence of an alternative neuromuscular, mechanical
or metabolic explanation for hypoventilation. Daytime Hypercapnia is the hallmark of OHS,
differentiating it from OSA alone. Although OHS/Pickwickian syndrome could have a
component of hypoxia coexistent in the more severe cases, it is not required for defining
OHS. Other causes of hypoventilation have to be ruled out before making the final
diagnosis. Obesity is required at a range of BMI greater than 30.
A 57-year-old female was admitted to the medical intensive care unit (MICU) for septic
shock and acute respiratory failure requiring endotracheal intubation secondary to
multilobar pneumonia. She was started on appropriate therapy; however, her MICU course
was complicated with the development of intermittent rapid, narrow complex tachycardia
episodes that usually resolve with vagal maneuvers; however, it is not refractory. On
physical exam, vitals are blood pressure 110/75 mmHg, heart rate 180 bpm, respiratory
rate 18/minute, and oxygen saturation 90% on FiO2 of 70%. Her only access is a right
internal jugular triple lumen central line in which she is administered intravenous fluids
and antibiotics. Which of the following is the next step to treat the patient's tachycardia?
1. Start patient on esmolol drip starting at 50 mcg/kg/min and titrate accordingly
2. Administer adenosine at 6 mg IV bolus followed by 20 mL of a saline flush
3. Administer adenosine at 3 mg IV bolus followed by 20 mL of a saline flush
4. Load patient with digoxin at 10 mcg/kg IV over the next 24 hours
Answer: 3 - Administer adenosine at 3 mg IV bolus followed by 20 mL of a saline flush
Explanations: ACC/AHA/HRS guidelines state that first-line therapy for supraventricular
tachycardia (SVT) in an adult is vagal maneuvers or adenosine. Adenosine should be
administered at a lower dose when given through central access, starting at a dose of 3 mg
IV bolus. Other causes to decrease the initial dose to 3 mg are in cardiac transplant
patients or patients on dipyridamole or carbamazepine drugs. Any patient receiving
adenosine should be on a form of cardiac monitoring. Patients treated for SVT are often on
a 12-lead electrocardiogram rhythm monitoring to assess the underlying rhythm while
adenosine is actively affecting the atrioventricular (AV) node. Adenosine can be further
classified as a miscellaneous antiarrhythmic drug outside the Vaughan-Williams
classification scheme. It acts on receptors in the cardiac AV node, significantly reducing
conduction time through hyperpolarization.
A 44-year-old previously healthy female presents to the emergency department in septic
shock. Initial vitals are temperature 101.5 F, heart rate 115 bpm, blood pressure 110/50
mmHg, respiratory rate 24/minute, and pulse oximetry (SpO2) 78% on a non-rebreather
mask. She is intubated for hypoxemic respiratory failure and acute respiratory distress
syndrome secondary to pneumonia. She is placed on inverse ratio ventilation (IRV) to
improve her oxygenation. Over the next 5 minutes, her SpO2 steadily increases to 95%.
However, her repeat blood pressure has decreased to 70/40 mmHg. What is the most
likely etiology of her hemodynamic compromise?
1. Pneumothorax
2. Increased systemic inflammatory response syndrome (SIRS) response and vasodilation
secondary to inflammatory mediators released in the lungs in response to barotrauma
3. Distributive shock secondary to sepsis
4. Decreased venous return
Answer: 4 - Decreased venous return
Explanations: Hemodynamic compromise secondary to decreased venous return is a
known complication of inverse ratio ventilation (IRV). The increased mean airway
pressures used in IRV translate to increased intrathoracic pressure, which may cause an
iatrogenic obstructive shock picture. Patients with preexisting hemodynamic instability
are at increased risk for shock when placed on IRV. Patients with sepsis may have an
element of distributive shock that goes unrecognized due to compensation. However, if an
additional insult occurs, such as an obstruction to right ventricular preload, this shock may
rapidly decompensate.
A 30-year-old, previously good health, non-smoker, triathlete is helped out of the water
due to extreme exhaustion and dyspnea. While being examined, he begins coughing up
blood-tinged sputum. He is transported to the local emergency room on 100% oxygen due
to oxygen saturation of 91%. All other vitals are stable. What is expected on chest x-ray
when he arrives at the hospital?
1. Cardiomegaly
2. Unilateral pleural effusion
3. Widened mediastinum
4. Kerley B lines
Answer: 4 - Kerley B lines
Explanations: While there is some component of heart failure in immersion pulmonary
edema (IPE) it may or may not be significant enough to show up on chest x-ray. Pleural
effusions may be seen with IPE, but it would be bilateral. While there is a redistribution of
blood centrally during submersion which may lead to increased diameter of the inferior
vena cava, it may not be radiographically significant, and there is a rapid redistribution
after removal from the water. Kerley B lines are indicative of pulmonary edema.
During the winter, a patient presents with headache, nausea, vomiting, and dizziness. A
medical history reveals that the patient is homebound and conserving money by using his
fireplace to keep warm. An arterial blood gas reveals he has a decreased oxygen-carrying
capacity and a normal arterial PO2. To which gas has the patient most likely been
exposed?
1. Methane
2. Carbon monoxide
3. Nitrogen dioxide
4. Sulfur dioxide
Answer: 2 - Carbon monoxide
Explanations: Carbon monoxide (CO) can be produced whenever carbon-containing
materials are burned, especially when ventilation is inadequate. CO binds much more
readily to hemoglobin than oxygen (220:1) and carboxyhemoglobin is formed rather than
oxyhemoglobin. The oxygen-carrying capacity is diminished and will eventually lead to
tissue hypoxia. One hundred percent hyperbaric oxygen therapy is the treatment of
choice. Nitrogen dioxide, sulfur dioxide, and methane can all cause hypoxia but do not
affect the oxygen-carrying capacity of blood. Nitrogen dioxide and sulfur dioxide are direct
respiratory mucous membrane irritants. Methane, an asphyxiant, simply decreases the
amount of oxygen available for oxygenation.
A 28-year-old female presents with a pleural effusion and undergoes ultrasound-guided
thoracentesis. The fluid collected is sent for cytopathologic examination, and a population
of atypical cells is identified. Which of the following immunohistochemical markers, if
positive, likely indicate the cells are of mesothelial origin?
1. MOC-31
2. Ber-EP4
3. Calretinin
4. CDX2
Answer: 3 - Calretinin
Explanations: Distinguishing reactive mesothelial cells from malignant/neoplastic
populations of cells in pleural effusion analysis can be difficult and utilizing
immunohistochemical (IHC) markers can aid in the diagnosis and differentiation of
mesothelial proliferation. Calretinin, D2-40, and cytokeratin 5/6 are among the most
important IHC markers to examine when determining if mesothelial cells are present in
pleural effusion samples. CDX2 is a marker for gastrointestinal adenocarcinoma and is a
useful IHC to differentiate between malignant mesothelioma and metastatic
gastrointestinal adenocarcinoma. Ber-EP4, MOC-31, and CEA are markers which if positive
likely indicate carcinoma cells in pleural effusion.
An adult school bus driver presents with a one-week history of a pruritic, vesicular rash
and a recent development of dyspnea and coughing. Physical exam finds new and old skin
lesions, and a chest x-ray confirms a nodular infiltrate. Which is the most probable
causative organism?
1. Mycoplasma pneumoniae
2. Epstein-Barr virus
3. Varicella-zoster virus
4. Rhinovirus
Answer: 3 - Varicella-zoster virus
Explanations: The case of the rash with vesicles in varying stages of development and subsequent
pneumonia suggests varicella-zoster. Varicella pneumonia typically develops between 3 and 7 days
following the rash, and, in adults, it is the leading cause of mortality and morbidity from the varicella
virus. The prodromal symptoms in adolescents and adults are myalgia, nausea, decreased appetite,
and headache followed by a rash, oral sores, malaise, and a low-grade fever. Oral manifestations may
precede the skin rash. In adults, the rash may be more widespread, fever may last longer, and they
are more likely to develop pneumonia. Because watery nasal discharge containing live virus precedes
exanthems by 1 to 2 days, the infected person becomes contagious 1 to 2 days before diagnosis. The
goal of treatment is symptom relief. As a protective measure, those infected are usually required to
stay at home while they are infectious. Cutting the nails short or wearing gloves may prevent
scratching and the risk of secondary infections. Topical calamine lotion may relieve pruritus. Daily
cleaning of the skin with warm water will help avoid secondary bacterial infection. In adults, infection
tends to be more severe and treatment with antiviral drugs, such as acyclovir or valacyclovir, is
advised if started within 24 to 48 hours of rash onset. Adults should be advised to increase water
intake to reduce dehydration. Acetaminophen is recommended for headache. Antihistamines relieve
itching. Those at risk of developing complications or those with significant exposure may be given
intramuscular varicella-zoster immune globulin, a preparation containing high titers of antibodies to
the varicella-zoster virus, to help prevent the disease. Sorivudine, a nucleoside analog, may be an
effective treatment for primary varicella in healthy adults.
A patient presents to the clinic and complains of photophobia, has light skin, and silvery
hair. He appears to be an albino and complains of suffering frequent bouts of sinusitis,
pneumonia, and acne. He was diagnosed with neuropathy at age 16, but no one has ever
diagnosed the cause. What syndrome do you suspect might be responsible for his
symptoms?
1. Sezary Disease
2. Caplan Syndrome
3. Felty Syndrome
4. Chediak-Higashi Syndrome
Answer: 4 - Chediak-Higashi Syndrome
Explanations: Chediak-Higashi Syndrome typically presents with clotting problems, leaving
the patient prone to easy bruising. Chediak-Higashi Syndrome causes neuropathy, usually
diagnosed when the patient is a teenager. Chediak-Higashi Syndrome causes impaired
phagocytosis of bacteria, which impairs bacteriolysis, leading to frequent infections of the
skin, mucous membranes, and respiratory tract from bacterial infections. People with
Chediak Higashi Syndrome suffer from oculocutaneous albinism. They typically have light
pigmentation of the skin, hair, and eyes, and frequently complain of photophobia and
sensitivity to light.
A 60-year-old male patient presents with severe multilevel spondylosis presents with
progressive weakens of his upper extremities. He is not able to lift his both hand above the
shoulder and not able to do any fine activities with his hands. For the past 4 weeks, his
weakness has progressed significantly. He also reports progressive exertional
breathlessness. His reports that when he lays down flat, he gets choking sensation and has
to get up from the bed. His MRI of spine shows severe multilevel cervical cord
compression. Chest x-ray showed elevated left diaphragm. What is the next best step in
the evaluation of this patient?
1. MRI of the brain to look for intracranial pathology as strokes are common in this patient
population
2. Echocardiogram to rule out congestive heart failure in view of orthopnea
3. Polysomnography to rule out sleep apnea as patient reports symptoms in the night
4. Ultrasound of thorax to look for diaphragmatic movements with deep breathing to rule
out diaphragmatic palsy
Answer: 4 - Ultrasound of thorax to look for diaphragmatic movements with deep
breathing to rule out diaphragmatic palsy
Explanations: Diaphragmatic palsy is usually detected incidentally on routine chest x-ray as
an elevated diaphragm. Normally the dome of the right diaphragm is at the level of the
anterior end of the fifth rib, and the left diaphragm is one intercostal space lower than
that. All patient with suspected diaphragmatic palsy should undergo fluoroscopic
evaluation to look for the caudal and outwards movements of the diaphragm with deep
inspiration or sniffing. In the recent times, ultrasound of the chest is increasing used to
detect diaphragmatic movements. It can be done bedside and avoids the radiation
exposure of fluoroscopic examination. The diaphragm appears as a thin hypoechoic line
when viewed through the subhepatic or sub splenic view on b mode. On deep inspiration,
diaphragm contacts and moves towered abdomen. This can be easily measured and
tracked in the M mode, and the diaphragmatic excursion can be measured. In severe
paralysis, the normal diaphragmatic contraction on the opposite side will create negative
pressure and sucks the diaphragm into the thorax during the inspiration causing
paradoxical cranial movements of the diaphragm with inspiration.
A 42-year-old man was admitted two days back after an intentional overdose of his
medications at home. His chronic medical conditions include hypertension, mild
pulmonary hypertension, and Raynaud syndrome. He does not smoke or drink alcohol.
Initial presentation was with acute shock and hypoxic respiratory failure requiring
emergent endotracheal intubation. Initial lab work did not show any renal or hepatic
dysfunction. He has since been on vasopressors with maximal norepinephrine dose of 10
micrograms per minute 24 hours back and slowly being weaned. He has also received
insulin and dextrose infusion and several grams of calcium chloride within the first 24
hours. On the third day, the nurse noticed new skin rashes over extremities. He does not
have any fever. Bloodwork shows normal coagulation parameters and platelets. However,
glomerular filtration rate shows an acute drop to 32. In addition, transaminases are four
times elevated from baseline. To what could one attribute these changes in his clinical
picture?
1. Progression of shock and multiorgan failure
2. Vasopressor toxicity with poor end-organ perfusion
3. Calcium chloride effect
4. Acute leptospirosis
Answer: 3 - Calcium chloride effect
Explanations: The patient likely has a calcium channel blocker overdose, as his medical
conditions of hypertension and Raynaud syndrome require treatment with a calcium
channel blocker agent. He has no fever or leukocytosis and had normal renal and liver
function. This is therefore unlikely an infection like leptospirosis Calcium chloride or
gluconate are often used in the initial treatment of calcium channel blocker overdose.
Overaggressive use in the first 24 hours is often seen and can rarely result in calciphylaxis
acutely presenting with skin necrosis, rash, acute kidney injury, and abnormal liver
function. Progression of multiorgan failure or vasopressor adverse effects is unlikely to
develop now when his needs are slowly improving. Although hemodialysis is not indicated
in the treatment of calcium channel blocker toxicity, it may be indicated with worsening
toxicity of calciphylaxis and kidney injury in the setting of calcium channel blocker
overdose.
A 65-year-old male presented with symptoms of chronic cough for 1 month with recurrent
episodes of bloody sputum. He also complains of a 10-pound (4.5 kg) weight loss over the
past 2 months. He has also been having some difficulty in swallowing lately. A CT scan and
x-ray of the chest shows a cavitary lung lesion in the right upper lobe. He was admitted to
the hospital and started empirically on piperacillin/tazobactam and levofloxacin. Sputum
cultures are sent on 3 separate occasions which come back as negative, but a
galactomannan antigen assay done after 2 days came back as positive. Which of the
following is the most appropriate next step in the management of this patient?
1. Bronchoscopy and biopsy
2. Continue the same regimen with swallow evaluation and maintenance of aspiration
precautions
3. Start voriconazole with positive galactomannan
4. Consider right upper lobe wedge resection
Answer: 1 - Bronchoscopy and biopsy
Explanations: Assay against galactomannan antigen, a polysaccharide component of the
cell wall has a high specificity for diagnosing aspergillus disease. However false positives
are seen in patients concurrently on piperacillin/tazobactam. With ongoing weight loss,
hemoptysis, and cavitary mass lesions, the workup for malignant lesions should be
completed by bronchoscopy and transbronchial biopsy. A confirmation of aspergillus
species in fungal stain, culture, or polymerase chain reaction, adds weight to the diagnosis
but is not diagnostic by itself. Occasionally biopsy through bronchoscopy or video
thoracoscopy can also help in reaching the diagnosis.
A 50-year-old male with a history of chronic obstructive pulmonary disease (COPD) was
admitted for acute hypercapnic and hypoxic respiratory failure secondary to COPD
exacerbation. He developed severe acidosis from hypercapnia and required immediate
intubation and admission to the intensive care unit. On the second day of hospitalization,
he was improving, and plans were made to start a weaning trial the next morning.
Overnight the ventilator starts sounding alarms. When the clinician arrives, he sees that
the ventilator shows a peak pressure of 58 and a plateau pressure of 26. A physical exam
reveals bilateral breath sounds and good air movement. There is no engorgement of the
jugular veins, and the blood pressure is within normal limits. What are the most likely
diagnosis and the next step in management?
1. Acute mucous plugging or kink in the airway; suction the patient and assess airway
2. Worsening COPD; give a pulse dose of corticosteroids and start continuous nebulizer
treatment
3. Tension pneumothorax; place a needle in the second intercostal space followed by a
chest tube
4. Air trapping secondary to auto-PEEP; adjust the I:E (inspiratory: expiratory) ratio
Answer: 1 - Acute mucous plugging or kink in the airway; suction the patient and assess
airway
Explanations: An increase in the peak pressure without change in plateau pressures is
indicative of increased airway pressures, which is likely secondary to a mucous plug, the
patient biting down on the tube, or the tube being kinked. Initial management is to
evaluate the airway and suction the patient for possible mucous plug(s). Worsening
chronic obstructive pulmonary disease would likely present as decreased breath sounds
bilaterally and poor air movement. Also, a synchronous increase in plateau pressure would
be expected. Tension pneumothorax would present with hypotension, hypoxia, and
increased plateau pressure. One would expect for the patient to have decreased breath
sounds unilaterally and to have jugular venous distention. Auto-PEEP with air trapping
causes an increase in both peak and plateau pressures. Patients frequently seem to be
dyssynchronous with the ventilator.
A 16-year-old white female presents to the emergency department with an acute
exacerbation of shortness of breath, chest tightness, and wheezing that has not responded
to her inhalers at home for over 4 hours. She is using accessory muscles to breathe and
can only speak in one or two words. She mentions that she is tired. The difference in
systolic blood pressure between inspiration and expiration is 10%. She needs 2 liters of
oxygen to maintain saturation at 94%. She is diffusely wheezy but does not have any
secretions. Which of the following treatment strategies is least likely to benefit her
current disease process?
1. Heliox with 40:60 parts of helium and oxygen
2. Use of BIPAP support
3. 125 mg of methylprednisolone followed by 60 mg every 6 hourly
4. 2 separate doses of 2 g magnesium sulfate IV an hour apart
Answer: 1 - Heliox with 40:60 parts of helium and oxygen
Explanations: The patient appears to be in status asthmaticus. She is somewhat fatigued
but still awake. She does not have any significant secretions. Treatment with noninvasive
ventilatory support is appropriate as is methylprednisolone and magnesium at the
mentioned dose. Heliox is rarely used in status asthmaticus in patients with definitive
evidence of pulsus paradoxus with upper airway wheeze or stridor. Its ratio is 70:30 or
60:40 for oxygen to helium. The patient has no pulsus paradoxus which is interpreted as
greater than 12% variation in systolic blood pressure with respiration. Heliox is avoided
due to its prohibitive cost, infrequent indication and need for recalibration of gas blenders
and flow meters when used with mechanical ventilation.
A patient is postoperative day 4 following a right pneumonectomy. He is afebrile,
hemodynamically stable, and has a normal white blood cell count. A chest x-ray reveals a
newly formed air-fluid level. What is the appropriate management at this time?
1. Medical management with intravenous antibiotics
2. Revision of his stump via a median sternotomy
3. Revision and reinforcement of his stump via the original thoracotomy
4. Perform bronchoscopy
Answer: 4 - Perform bronchoscopy
Explanations: Initial therapy for a bronchopleural fistula requires immediate chest tube
placement and bronchoscopy. Subsequent therapy depends on the time of occurrence of
the fistula. Those bronchopleural fistulas occurring within 1 week of surgery usually are
due to technical failure, have minimal contamination, and the bronchus usually can be
closed with a buttressed muscle flap. Fistulas that occur later than 10 days usually are
associated with an empyema. Immediate surgery is not recommended. Tube
thoracostomy, antibiotics, and jet ventilation may be required if the air leak is significant.
The closed tube drainage can later be converted to an open window followed by a Clagett
procedure. The above patient is asymptomatic, but a new air-fluid level should raise the
possibility of a bronchopleural fistula. The patient needs a bronchoscopy, although at
times a fistula may not be evident with this approach. Occasionally, dye has to be injected
into the chest cavity, and bronchoscopy repeated. At this moment, the patient should be
closely watched, and a chest tube set should be placed at the bedside. However, if
definitive bronchial dehiscence is observed, he should return to the operating room for
revision of his bronchial stump with a possible muscle flap.
A 27-year-old woman is being treated for active pulmonary tuberculosis. Treatment
includes daily isoniazid, rifampicin, ethambutol, and pyrazinamide. Three weeks into
treatment, she is found to be HIV positive with a CD4 count of 414 cells/mm3. Her vital
signs are normal, and the physical examination is unremarkable. Pending initiation of HIV
therapy, which of the following is the most appropriate management for this patient?
1. Add streptomycin to the regimen
2. Switch to a thrice-weekly regimen
3. Add gentamicin to the regimen
4. No change is necessary
Answer: 4 - No change is necessary
Explanations: First-line treatment for pulmonary tuberculosis (TB) occurs in two phases. In
the intensive phase, the regimen consists of isoniazid, rifampicin, ethambutol, and
pyrazinamide daily for two months. No change in the drug regimen is necessary for this
patient. The intensive phase is followed by isoniazid and rifampicin daily for four months.
The continuation phase can be extended in patients with HIV who are not receiving
antiretroviral therapy (ART) during TB treatment. Streptomycin has fallen out of favor as a
first-line agent in the treatment of TB.
An elderly male with long-standing chronic obstructive pulmonary disease (COPD) is
rushed to the emergency room with exacerbation of his COPD. He is severely short of
breath and gasping. Quick blood work reveals an arterial blood gas of pH 7.15, PCO2 55,
PO2 54 and HCO3 15. He is admitted to the intensive care unit and the pulmonologist
intubates him. He remains on the mechanical ventilator for 13 days and is eventually
extubated. According to the 2015 Global Initiative for Chronic Obstructive Lung Disease,
what is the mortality of such patients at 12 months after discharge?
1. Less than 5%
2. 10% to 15%
3. 40% to 60%
4. 90%
Answer: 3 - 40% to 60%
Explanations: COPD carries a high morbidity and mortality. The GOLD report indicates that
patients with COPD who require mechanical ventilation have a high mortality at 12
months. Any acute exacerbation should be promptly treated because delays often result in
protracted hospital admissions. The aim is to reduce the burden of COPD. Factors
associated with negative prognosis include hypercapnia, low body mass indices, advanced
age, low serum albumin, long-term use of corticosteroids and poor functional status.
A 45-year-old man presented to the emergency room because of acute right-sided chest
pain. The pain started while he was moving furniture and improved by rest. He has a past
medical history of atrial fibrillation on rate controlled with metoprolol and diabetes on
sitagliptin, with no significant family history. Initial vital signs showed blood pressure of
126/81 mm/Hg; pulse rate was 85 beats/min, oxygen saturation was 94% on room air.
Physical examination revealed bilateral air entry, regular S1, S2 with no added sounds or
murmurs. Initial labs showed troponin of 0.03 with Creatinine of 1.7. EKG showed atrial
fibrillation. He was admitted for further evaluation and management. Subsequent
troponin at 6 and 12 hours were negative, and he was discharged home. On follow up with
his primary care provider, the medical resident decided to proceed with CT coronary
angiography, but the attending thinks it is an inappropriate next step. What was the
attending concern?
1. CT angiography is contraindicated with atrial fibrillation.
2. He has a high risk and should go for cardiac cath.
3. He has Creatinine of 1.7
4. He has a history of diabetes
Answer: 3 - He has Creatinine of 1.7
Explanations: After acute coronary syndrome has been ruled out, the clinician has an
abundance of diagnostic options to choose from when aiming to determine the presence
of CAD and quantify its extent in these patients. Coronary computed tomographic
angiography (CCTA) is an anatomic test that can be used in intermediate-risk patients to
provide a diagnostician with these answers quickly. Absolute contraindications are a
patient history of severe or anaphylactic reaction to iodinated contrast, inability to
cooperate with scan protocols, hemodynamic instability, decompensated heart failure,
acute myocardial infarction, and renal impairment. Contrast-induced acute kidney injury
(CI-AKI) is the acute impairment of renal function further to the intravascular
administration of iodinated contrast media and occurs most frequently after coronary
angiography, percutaneous coronary intervention, and contrast-enhanced computed
tomography. CI-AKI has been associated with the development of acute renal failure,
worsening of chronic kidney disease, the requirement for dialysis, prolonged hospital stay,
and higher mortality rates and health care costs. Whole-heart CT enables evaluation of
coronary arteries with high image quality, low radiation exposure, and high diagnostic
accuracy in patients with chronic atrial fibrillation, with a diagnostic performance similar to
that in patients with sinus rhythm.
A 68-year-old female presents for complaints of 4-month left-sided chest pressure and
shortness of breath. She has a significant medical history of gastroesophageal reflux
(GERD), chronic obstructive pulmonary disease (COPD), and hypertension. The patient
describes her symptoms are worse with exertion and relieved with rest. Her current
medications are lisinopril, hydrochlorothiazide, albuterol, and combination inhaled
corticosteroid/long-acting beta agonist. On physical exam, vitals are blood pressure 140/80
mmHg, heart rate 90 bpm, respiratory rate 20/minute, and oxygen saturation 94% on
room air. Lung exam is significant for bilateral end-expiratory wheezes throughout all lung
fields. ECG is significant for left axis deviation and left ventricular hypertrophy (LVH). The
patient is planned for a cardiac stress test. Which of the following diagnostic tests is
contraindicated in this patient?
1. Coronary catheterization
2. Dobutamine stress echocardiography
3. Exercise stress echocardiogram
4. Adenosine myocardial perfusion imaging
Answer: 4 - Adenosine myocardial perfusion imaging
Explanations: Adenosine is a vasodilator that can cause bronchospasm. Active wheezing in
asthma and COPD are contraindications for pharmacologic vasodilators. Adenosine and
other pharmacologic vasodilators assess significant coronary artery disease by a
mechanism called coronary steal. Other vasodilators such as regadenoson and
dipyridamole, like adenosine, are contraindicated in patients who are actively wheezing as
they can cause bronchospasm. Adenosine has a very short half-life, and adverse effects are
usually transient and short-lived. However, bronchospasm can cause significant and fatal
consequences and should be avoided in patients with active wheezing.
A 55-year-old male with a history of chronic obstructive pulmonary disease and
bronchiectasis, a current everyday smoker with a 50 pack-year history, was brought into
the emergency department due to unresponsiveness. His wife confirms that the patient
was not feeling well for the past four days with an upper respiratory viral infection. His
symptoms included generalized malaise, fevers, shortness of breath, and cough. His home
medications include fluticasone and salmeterol twice a day and albuterol as needed for
shortness of breath. He went to urgent care two days ago and was prescribed
azithromycin, prednisone, albuterol, and ipratropium nebulization which only provided
temporary relief. In the emergency department, vital signs show a blood pressure of
140/72 mmHg, heart rate of 98/min, respiratory rate of 10/min, temperature 38.3 C, and
pulse oximetry of 88% on room air. On physical examination, the patient is noted to be
lethargic, mildly responsive to painful stimuli, and has decreased air movement on
auscultation of his chest. Arterial blood gas (ABG) was drawn, and the results are as
follows, pH 7.02, pO2 55 mmHg, pCO2 100 mmHg. Chest X-ray showed bilateral multifocal
infiltrates. The patient was endotracheally intubated and placed on mechanical ventilation.
Initial settings were a respiratory rate of 20/min, tidal volume 500 m, a fraction of inspired
oxygen 1.0, and positive end-expiratory pressure of 5 cm H20. He is started on inhaled
bronchodilators, intravenous methylprednisolone, broad-spectrum antibiotics, and
oseltamivir.
He becomes asynchronous with the ventilator requiring sedation and later neuromuscular
blockade. His repeat ABG shows a pH of 6.95, pO2 70 mmHg, pCO2 120 mmHg, and he is
currently becoming hypotensive. Due to difficulty with ventilation, the patient is placed on
a venovenous extracorporeal membrane oxygenator (VV-ECMO). The patient is cannulated
for VV-ECMO. The inflow cannula is inserted in the right femoral vein and outflow cannula
in the right internal jugular vein with flows of 3 liters per minute, sweep four, and fraction
of inspired oxygen 1.0. ABG on these settings show pH 7.20, pO2 50 mmHg, pCO2 70
mmHg. What is the next best step in management?
1. Start the patient on inhaled nitric oxide
2. Increase the dose and frequency of intravenous steroids
3. Order a stat chest ultrasound
4. Increase flow to 4 liters/minute
Answer: 3 - Order a stat chest ultrasound
Explanations: It is possible to perform VV-ECMO cannulation blindly, but it is
recommended to cannulate for ECMO under direct visualization, usually with fluoroscopy
or ultrasound, to confirm appropriate placement of the inflow and outflow cannulas. In
the double VV-ECMO cannulation, there are two ways to insert the cannulas. The inflow
cannula is usually inserted in the femoral vein and advanced to the junction of the inferior
vena cava (IVC), and the right atrium and the outflow cannula can be inserted into the
right internal jugular vein and advanced through the superior vena cava to the right
atrium. The second approach is to insert the inflow cannula into the femoral vein and
advanced to the mid-IVC, and the outflow cannula can be inserted in the contralateral
femoral vein with the tip ending in the right atrium. The correct placement of the cannulas
is important to avoid the recirculation phenomenon. This is when the oxygenated blood
that is being returned to the patient is drained by the inflow cannula before being
circulated through the body. This is usually due to the two cannulas being in close
proximity. The flow rate, the fraction of inspired oxygen, the amount of recirculation of
blood will all affect the ability of the VV-ECMO circuit to oxygenate the blood. Increasing
the flow when pO2 is low is appropriate, but the placement of the cannulas should always
be confirmed after cannulation.
A veterinarian presents with one day of flu-like symptoms after visiting a rural cattle farm
three days ago for a first-time evaluation. The farmer had reported that some animals had
appeared ill, but diagnostic testing on the animals and soil are still pending. A chest x-ray
on the patient is obtained and shows a widened mediastinum with hilar adenopathy. The
patient has an oral temperature of 100.9 F, heart rate 105 bpm, respiratory rate
18/minute, and oxygen saturation 96% on room air. The patient denies weight loss,
hemoptysis, hematochezia, or recent known sick contacts. Which of the following is the
most likely diagnosis?
1. Boerhaave syndrome
2. Chest lymphoma
3. Descending necrotizing mediastinitis
4. Respiratory anthrax
Answer: 4 - Respiratory anthrax
Explanations: While Boerhaave syndrome has an acute onset with the development of a widened
mediastinum, this patient presentation does not fit. This condition is usually associated with
vomiting and a sudden rise in intraluminal esophageal pressure causing the tear. Some physical
exam findings may be Hamman crunch, hemoptysis, and chest pain. However, esophageal
rupture in Boerhaave syndrome is particularly uncommon and does not present with flu-like
symptoms after exposure to cattle or soil. Chest lymphoma is the most common cause of
widened mediastinum. However, this patient has not reported weight loss and no prior history of
illness is reported in the question stem. This makes this diagnosis less likely given the acute
nature of symptoms. Descending necrotizing mediastinitis is a rare diagnosis but a potentially
fatal cause of a widened mediastinum. This condition is usually caused by the spread of infection
from a primary head, oral, or neck infection that progresses to the mediastinum. These infections
are usually polymicrobial and gas producing which contributes to the development of a widened
mediastinum. Inhalation anthrax is also rare in the United States, and no cases have been
reported. However, a high index of suspicion should be maintained given the potential for
weaponization. This question stem includes multiple historical features that should trigger a
concern for inhalation anthrax including rural setting, cattle exposure, and occupational hazard
(veterinarian). Anthrax results in a prodromal viral respiratory illness that lasts about 1week,
followed by acute hypoxia, dyspnea, and acute respiratory distress syndrome. In some patients,
mediastinal widening and hilar adenopathy are seen on x-ray.
A 62-year-old male is referred by his primary care provider to the pulmonary clinic for
evaluation of dyspnea on exertion. He can walk two to three city blocks without dyspnea
but lately has been dyspneic when walking up hills or climbing stairs. He is a nonsmoker
and worked in coal mines from 20 to 25 years of age. Since then, he has been a
schoolteacher. He has no history of weight loss or night sweats and no family history of any
type of cancer. His father had emphysema diagnosed at 50 years of age and mother at 62
years of age. His father was a heavy smoker and worked in coal mines. His primary care
office sent his echocardiogram and chest x-ray reports. The echocardiogram shows an
ejection fraction of 50%, with mild elevation of pulmonary artery pressures at 30 mmHg.
The chest x-ray shows emphysema in both lower lobes. What are the best next tests to
order?
1. Pulmonary function tests (PFTs) and right heart catheterization
2. PFTs and left heart catheterization
3. CT chest with pulmonary embolism protocol
4. PFTs and alpha anti-trypsin level
Answer: 4 - PFTs and alpha anti-trypsin level
Explanations: Pulmonary function tests (PFTs) and alpha anti-trypsin levels should be
ordered next. The patient likely has emphysema induced by low antitrypsin levels, causing
his dyspnea on exertion. PFTs will show obstructive lung disease with a low diffusing
capacity of the lungs for carbon monoxide (DLCO). Low alpha anti-trypsin levels affect the
lower parts of lungs. This is why the chest x-ray showed emphysema in both lower lobes.
Respiratory causes of dyspnea on exertion include asthma, acute exacerbation of asthma,
chronic obstructive pulmonary disease (COPD), acute exacerbation of COPD, pneumonia,
pulmonary embolism, lung malignancy, pneumothorax, or aspiration.
Question 63: A 48-year-old male presents to the emergency department with two-and-a-half months
of progressively worsening left-sided chest pains and shortness of breath with associated low-grade
fevers, chills, occasional night sweats, productive cough of foul-tasting sputum, and 15 lbs.
unintentional weight loss. He does not have a primary care provider, denies any medical or surgical
history, does not believe in "Big Pharma" and takes no medications. He denies any sick contacts,
refuses all vaccinations, does not drink any alcohol, use any tobacco products or illicit drugs, but does
report an extensive travel history in South Asia over the last two years as a Christian missionary. He
reports that he got home about six weeks ago from Vietnam, but his symptoms started off milder
probably 6 to 8 weeks before that. He states they seemed to coincide a week or so after he had
gotten over a really bad case of food allergies. He was not sure what he ate that set it off but
reported a two-week history of very high fevers, abdominal pains, and cramping with nausea,
vomiting, loss of appetite, and generalized itching that gradually subsided. He did seek medical
treatment at one point, but only got intravenous fluids, an antihistamine, and antiemetic medication.
They did not do any blood work or imaging. He reports feeling better for about a week before the
new symptoms started. He was busy preparing for his trip back to America, so he did not pursue any
medical care until now. He comes in tonight because he can't take the fevers and night sweats any
more which woke him up once again from sleep. He does admit that he searched his symptoms on
the internet last week and is worried that he might have cancer. Vitals signs are temperature 38.4 C,
heart rate 108 bpm, respiratory rate 24/minute, blood pressure 112/67 mmHg, and oxygen
saturation 86% on room air. He appears acutely ill, no acute distress and is normocephalic and
atraumatic. Sclera are nonicteric and pupils' equal round and reactive to light and accommodation.
The neck nontender with no jugular venous distention. He is mildly tachycardic with regular rhythm
and no murmur. Lungs are clear to auscultation bilaterally. The abdomen is soft, nontender with
normal bowel sounds. There is no costovertebral angle tenderness or bladder distension. He has
normal speech with no neurologic deficits. His chemistry panel shows normal electrolytes, normal
renal function, and normal hepatic function. His complete blood count shows mild leukocytosis with
WBC count of 11.8/microL and differential reveals 6.8% eosinophils. His chest x-ray shows a there is a
large cavitary lung lesion in the left upper lobe. CT scan of the chest shows there is a 4 cm x 6 cm ring-
enhancing cavitary lung abscess in the left upper lobe with air-fluid level. There are no malignant
findings. The patient is diagnosed with sepsis secondary to a lung abscess. He is admitted to the
medical floor and started on broad-spectrum antibiotics with intravenous vancomycin and
intravenous ampicillin-sulbactam. Over the next three days, the patient continues to fever and have
persistent symptoms. He undergoes bronchoscopy and bronchoalveolar lavage on hospital day four.
Cytology of the fluid is negative for malignancy but does note a high preponderance of eosinophils
and trematode eggs that are yet to be characterized. His blood and sputum cultures remain negative
since admission. Based on the above clinical scenario, what is the most likely causative organism?
1. Taenia solium
2. Enterobius vermicularis
3. Diphyllobothrium latum
4. Fasciola giganticum
Answer: 4 - Fasciola giganticum
Explanations: Ectopic fascioliasis is very rare, but the parasite may present in nearly any
area of the body and has been described in the literature. While Fasciola gigantica go
almost anywhere, Enterobius vermicularis and Diphyllobothrium latum stay primarily in
the gastrointestinal tract. Taenia solium has been known to cause primarily central
nervous system lesions from larval migration. Peripheral eosinophilia in varying degrees is
common amongst all filarial worm infections and is nonspecific but should clue one into a
parasite as the culprit. Fasciola giganticum infection is more commonly associated with
travel to Asia, the Pacific Islands, and some parts of Northern Africa than any of the other
species listed. The patient had spent the last two years in South Asia as a missionary and
had just returned. The patient had complained of a severe gastrointestinal illness lasting a
week or two that preceded his current illness. This likely signified the acute phase of the
fasciola infection when metacercariae migrate out of the intestines and through the liver
causing an overwhelming inflammatory and immune response.
A 12-year-old female presents to the emergency department in severe respiratory distress.
On lung auscultation, bilateral rhonchi and coarse breath sounds are appreciated. The
patient appears toxic, and her vital signs show a temperature of 103.7 F, pulse 152/min,
blood pressure 107/73 mmHg, and oxygen saturation 87% on room air. A STAT portable
chest x-ray is consistent with ARDS. Per the patient's mother, she was recently treated with
penicillin for tonsillitis. She is up to date with her vaccinations and the mother denies
additional past medical history. Given the patient's presentation and most likely diagnosis,
which organism was the most likely cause of her tonsillitis?
1. Rhinovirus
2. Diphtheria
3. Fusobacterium necrophorum
4. Group A strep
Answer: 3 - Fusobacterium necrophorum
Explanations: The patient is presenting with Lemierre disease, a rare complication of
tonsillitis. Lemierre disease should be considered in patients who have been diagnosed
with tonsillitis and are unresponsive to penicillin. She meets SIRS criteria, and her chest x-
ray is significant for ARDS. This complication is most commonly preceded by tonsillitis
caused by Fusobacterium necrophorum. While Streptococcus tonsillitis can precede
Lemierre disease, it is not the most likely pathogen.
A 62-year-old male with a history of hypertension, diabetes mellitus, chronic obstructive
pulmonary disease (COPD) on 2L of home oxygen, congestive heart failure, and atrial
fibrillation on warfarin presents to the emergency department with shortness of breath.
His initial vitals are temperature 101.3 F, heart rate 121 bpm, blood pressure 150/90
mmHg, respiratory rate 24/minute, and pulse oximetry (SpO2) 80% on 10 L non-rebreather
mask. His exam is significant for rales in the left lower lung field and decreased air
movement diffusely as well as 1+ pitting edema to his lower extremities bilaterally. Chest
x-ray shows hyperinflated lungs, and a focal infiltrate in the left lower lobe. EKG shows
atrial fibrillation with a rapid ventricular response without signs of ischemia. He is
intubated for hypoxemic respiratory failure secondary to pneumonia. However, his
oxygenation does not improve with standard ventilator settings, and inverse ratio
ventilation (IRV)is being considered. Which of the following patient factors is most likely
to complicate this patient's management if he were to be placed on IRV?
1. Respiratory failure secondary to pneumonia
2. History of COPD
3. History of heart failure
4. Hemodynamic status
Answer: 2 - History of COPD
Explanations: This patient's history of severe COPD, indicated by his need for home
oxygen, decreased air movement on exam, and hyper-inflated lungs, put him at increased
risk for auto-positive end-expiratory pressure (PEEP) if placed on IRV. Typical management
of mechanical ventilation for patients with obstructive lung disease often involves
increasing the expiratory time to allow adequate exhalation time, not decreasing it as is
seen in IRV. Decreased expiratory time will lead to incomplete exhalation in this patient
with subsequently decreased ventilation and increasing intrathoracic pressures resulting in
barotrauma. In severe cases, intrathoracic pressure may continue to build with subsequent
breaths as each additional breath creates additional volume which is unable to be exhaled.
This may result in hemodynamic compromise or pneumothorax.
A 57-year-old male with alcoholic liver cirrhosis presents to the emergency department
after 2 days of hematemesis. He currently is not taking any medications. A previous chart
shows esophageal varices listed in the history. Today's CBC shows a hemoglobin of 5.8 g/dL
and hematocrit of 17.8%. The prothrombin time and INR are 22.5 and 1.7, respectively.
One unit of packed red blood cells and one unit of fresh frozen plasma are ordered to
transfuse. Within 3 hours of transfusion, the patient becomes dyspneic, and the lungs
sound very congested. His blood pressure is 95/62 mmHg and heart rate is 112 beats/min.
An emergent chest x-ray and arterial blood gas are ordered. What is the best next step?
1. Intubate the patient
2. Stop the transfusion and notify the blood bank
3. Continue the transfusion and give IV furosemide 40 mg emergently
4. Slow the rate of blood transfusion
Answer: 2 - Stop the transfusion and notify the blood bank
Explanations: The best treatment for transfusion-related acute lung injury (TRALI) is to
stop the transfusion. The blood transfusion will worsen the patient's condition. TRALI
usually is associated with plasma components such as platelets and fresh frozen plasma.
Supportive care is the mainstay of TRALI treatment. Oxygen supplementation is needed.
Also, IV fluids and vasopressors are needed for blood pressure support. Intubating the
patient and protecting the airway is next step if the patient's respiratory status worsens.
Diuretics should be avoided in TRALI treatment.
A 65-year-old male presents with a cough and hemoptysis for four weeks. CT showed a 1.5
cm right upper lobe lesion and normal-sized lymph nodes. PET-CT showed fludeoxyglucose
(FDG) uptake in the mass and right hilar lymph node. What is the next step in his care?
1. Do preoperative clearance for surgery
2. Refer to oncology
3. Perform convex probe endobronchial ultrasound sampling of hilar lymph node
4. Perform convex probe endobronchial ultrasound sampling of mediastinal and hilar
lymph nodes
Answer: 4 - Perform convex probe endobronchial ultrasound sampling of mediastinal and
hilar lymph nodes
Explanations: This is a high-risk patient for lung cancer. According to PET-CT, he has stage
IIB disease; therefore, surgery is an option. Before surgery, N2 and N3 diseases should be
ruled out. PET showed uptake in only an N1 node, but PET has a significant false-negative
rate. As per ACCP guidelines, N1 disease on PET should be confirmed with tissue biopsy.
The sampling of a hilar lymph node will only give a diagnosis but not staging. Preoperative
clearance is only done after ruling out metastatic and extensive disease.
A 65-year-old woman with a history of chronic obstructive pulmonary disease presents to
the emergency department with shortness of breath and increased sputum production.
She is treated with nebulized bronchodilator therapy and oxygen, with gradual
improvement of the dyspnea. Further review shows that she was admitted to the hospital
earlier this year with the same complaint and diagnosis. According to the American
College of Chest Physicians, which of the following will most likely prevent frequent
readmission to the hospital in this patient?
1. Long-term macrolide therapy
2. Long-term systemic corticosteroid therapy
3. Continuous oxygen therapy
4. Nebulized bronchodilator therapy
Answer: 1 - Long-term macrolide therapy
Explanations: Current guidelines by the American College of Chest Physicians recommend
long-term macrolide therapy be initiated in patients with moderate to severe
exacerbations of chronic obstructive pulmonary disease (COPD) after one or more
exacerbations in one year, despite being on optimal bronchodilator therapy. It is hoped
that prolonged antibiotic therapy will reduce hospitalizations and the frequency of COPD
exacerbations. COPD exacerbations can also be reduced by ensuring smoking cessation,
annual influenza vaccination, pneumococcal vaccination, and long-acting beta-agonist
therapy. Some patients may also benefit from pulmonary rehabilitation and avoidance of
triggering agents.
An 18-month-old, previously healthy child is admitted for pneumonia with empyema. After
fluid drainage using video-assisted thoracoscopic surgery, a 2 cm lung abscess is seen.
Which of the following would be the most appropriate therapy?
1. CT-guided needle drainage
2. Open thoracostomy urgently
3. Open thoracostomy after resolution of fever
4. Long-term intravenous antibiotics
Answer: 4 - Long-term intravenous antibiotics
Explanations: Most lung abscesses can be treated with long-term parenteral antibiotics.
Staphylococcus aureus is the primary cause of a lung abscess in children who are born and
raised in the United States. If methicillin-resistant S. aureus is determined to be the source
of a lung abscess, vancomycin and linezolid should be considered. Thoracostomy can result
in contamination of uninfected tissue.
A 17-year-old female with a past medical history of intravenous drug abuse who was
recently hospitalized one week ago for pneumonia presents to the emergency department
complaining of dyspnea, fever, chills, and a productive cough. She admits to not
completing her antibiotic therapy. Vital signs reveal a heart rate of 112/min, blood
pressure of 120/80 mmHg, respiratory rate of 26/min, and an oral temperature of 103.2 F.
Rigors are noted as well. Focused cardiopulmonary exam reveals sinus tachycardia, diffuse
rhonchi in bilateral lung fields, and decreased breath sounds at the left lung base. Imaging
reveals multifocal pneumonia with a large left-sided pleural effusion. Diagnostic
thoracocentesis is completed. Pleural fluid culture and sensitives are pending. Which
antibiotic will best target organisms with low-yield for positive cultures in pleural space?
1. Vancomycin
2. Metronidazole
3. Cefepime
4. Amikacin
Answer: 2 - Metronidazole
Explanations: In hospital-acquired empyema, it is imperative to cover for gram-positive
cocci, including methicillin-resistant Staphylococcus aureus (MRSA), especially in the
setting of this patient’s recent hospitalization and antibiotic exposure. Anaerobic bacteria
are notorious for an invasion of the pleural space, likely from its anaerobic environment,
but usually yield culture-negative media because they’re slow-growing organisms. When
selecting antibiotics for empyema, treatment must always include anaerobic coverage.
Metronidazole is an appropriate antibiotic for anaerobic coverage. Vancomycin & cefepime
are both appropriate choices for hospital-acquired empyema that covers gram-negative
organisms, including Pseudomonas and MRSA. However, these do not cover anaerobic
microorganisms.
A 17-year-old female presents for a three-day history of fever, chills, diarrhea, malaise, and
dyspnea. She has recently returned from a camping trip in Colorado, where she was
exposed to rodents in the campsite. Clinical examination shows a heart rate of
110/minute, respiratory rate of 25/minute, blood pressure of 100/70 mmHg, and
temperature of 101 F (38.2 C). A blood smear reveals marked leukocytosis with an
appearance of immunoblasts, which exceeds 10% of the total. A chest radiograph reveals
bilateral fluffy infiltrates. Which organism most likely is the cause of this presentation?
1. Chlamydia psittaci
2. Influenza virus
3. Streptococcus pneumoniae
4. Hantavirus
Answer: 4 - Hantavirus
Explanations: Hantavirus presenting as cardiopulmonary syndrome, such as in this patient,
is common in the Southwestern United States due to the particular genotypes found in the
US. Immunoblasts and thrombocytopenia are common, requiring hospital admission.
Hantavirus also is known to cause hemorrhagic fever with renal syndrome in Asia and
Europe and has more severe features of cardiac, hepatic, and renal failure. The virus
increases capillary permeability and decreases blood pressure causing an acute respiratory
distress syndrome-like picture with shock and potential renal failure. Hantavirus has an
incubation period of 7 to 28 days and presents with fever, chills, diarrhea, malaise, and
dyspnea. This is followed by hypotension, oliguria, and a characteristic diuresis in
hemorrhagic fever with renal syndrome type. Both syndromes are acquired from infected
rodents through urine or stool droppings. Rodent control is vital in preventing infections.
A 17-year-old male presents to the emergency department in Arizona with vomiting,
dyspnea, fever, and generalized myalgias. The vital signs demonstrate fever, tachypnea,
tachycardia, and mild hypoxia. The patient is found to have thrombocytopenia,
metamyelocytes, leukocytosis, and bilateral pulmonary infiltrates on chest x-ray. Which
organism is the most likely etiology?
1. Influenza A
2. Hantavirus
3. Human immunodeficiency virus (HIV)
4. Pneumocystis jiroveci
Answer: 2 - Hantavirus
Explanations: Hantavirus presenting as cardiopulmonary syndrome, such as in this patient,
is common in the Southwestern United States. Thrombocytopenia is the most common
significant laboratory abnormality on presentation when patients are reaching admission
criteria. Metamyelocytes may also be seen in the setting of significant leukocytosis.
Bilateral infiltrates seen on the chest x-ray may have an acute respiratory distress
syndrome appearance due to the fluffy nature due to increased capillary permeability.
Hantavirus often presents with fever, chills, diarrhea, malaise, and dyspnea. Hantavirus is
predominantly acquired through aerosolization of rodent urine and droppings.
A 25-year-old female comes to an outpatient clinic with a yearly cough which starts only
after a cold in late fall and early winter season and lasts until mid-summer. During this time
patient has severe coughing spells and the spells get better without treatment. She denies
any wheezing, paroxysmal nocturnal dyspnea, postnasal drip, allergies, gastroesophageal
reflux-like symptoms, and denies choking on food. Family history is noncontributory. She
does not have any pets or recent travel. She denies intravenous drugs or alcohol abuse and
is a nonsmoker. Physical examination is unremarkable. Blood workup, spirometry, and
chest x-ray are normal. What is the most likely cause of the cough?
1. Cough variant asthma
2. Aspiration
3. Chronic obstructive pulmonary disease
4. Sinusitis
Answer: 1 - Cough variant asthma
Explanations: Cough variant asthma presents primarily with coughing, not wheezing as in
typical asthma. These patients will have normal spirometry at baseline, but positive
methacholine challenge, when tested. This should be suspected if a cough is non-
productive, repetitive, occurs day and night, and is exacerbated by exercise, cold air or
upper respiratory infection. Look for positive family history or seasonal variation. This is
thought to be due to the fact that cough receptors are more prevalent in the proximal
airways, and decrease in density as the airways get smaller. The inflammation is more
prominent in the proximal airways where cough is stimulated, and less so distally, where
inflammation and narrowing would cause wheezing and dyspnea. Treatment is the same
for cough variant asthma as for typical asthma.
A patient with pulmonary alveolar proteinosis tested positive for anti-GM-CSF IgG
antibodies. He has significant dyspnea on exertion. He is depressed after his poor
performance on his pulmonary function tests. He asks if his lung function will ever return
to normal. What is true about pulmonary lung function testing and pulmonary alveolar
proteinosis?
1. A reduced diffusion capacity of around 40% to 50% is the most common finding
2. A reduced FEV1 to FVC ratio of around 60% to 70% is the most common finding
3. The alveolar-arterial gradient is typically less than 10 mmHg
4. Oxygen desaturation on a 6-minute walk test is only a late complication of pulmonary
alveolar proteinosis
Answer: 1 - A reduced diffusion capacity of around 40% to 50% is the most common
finding
Explanations: A restrictive pattern is the most common finding on spirometry in patients
with pulmonary alveolar proteinosis (PAP). Spirometry may show an airflow obstruction or
a mixed restriction and obstruction in patients who smoker or have underlying
emphysema. A reduced diffusion capacity is the most common finding on pulmonary
function testing and is often as low as 40% to 50%. Hypoxia is common in PAP and patients
may present with desaturations on a walk test or an elevated alveolar-arterial gradient
above 40 mmHg.
A 16-year-old white female presents to the emergency department with acute severe
worsening of her asthma symptoms that would not respond to home inhalers over the
past 6 hours. Medical history is significant for anxiety and bipolar disorder, irritable bowel
syndrome, hypothyroidism, GERD and a recent diagnosis of bronchial asthma. This is her
4th episode of severe asthma exacerbation in the past 4 months. Each one of them
required endotracheal intubation. She also gave birth to a healthy boy by a caesarian
section last year. Symptoms escalate rapidly in requiring intubation again. Her bloodwork
shows an absolute eosinophil count of 320/microL. Alarmed by these recurrent episodes
of severe asthma, a CT angiogram of the chest is ordered as well. It does not show
pulmonary embolism or chronic bronchiectasis. What further investigations may be most
helpful in treating her condition?
1. Send aspergillus specific antibody and IgE level
2. Send 5-hydroxy indole acetic acid level and chromogranin assay
3. Perform bronchoscopy when extubated
4. Speech therapy evaluation once extubated
Answer: 3 - Perform bronchoscopy when extubated
Explanations: Recurrent status asthmaticus requiring and reversed by endotracheal
intubation should always raise concern for tracheal pathologies like tracheomalacia or
excessive dynamic airway collapse (EDAC) which can be diagnosed by evaluation of upper
airway post-extubation. Risk factors for EDAC include prior intubation, GERD, chronic
respiratory inflammation, and thyroid disease. CT scan may appear normal particularly if
done in the inspiratory phase. Treatment once diagnosed may involve placement of a
tracheal stent. Evaluation by speech therapy can be effective for pure vocal cord
dysfunction once tracheal collapse or tracheomalacia has been ruled out by airway
evaluation.
A patient is being admitted to the hospital for further evaluation due to concern for
Hantavirus pulmonary syndrome after presenting with dyspnea, vomiting, and fever. What
laboratory abnormality is most significant on presentation demonstrating the need for
admission?
1. Thrombocytopenia
2. Hypernatremia
3. Hypokalemia
4. Leukocytosis
Answer: 1 - Thrombocytopenia
Explanations: Thrombocytopenia is often present at the stage of disease requiring
admission for further observation. This often precedes the descent into pulmonary failure
by a couple of days. Hyponatremia is more common than hypernatremia in Hantavirus
pulmonary syndrome. Potassium is not normally greatly affected in the early stages of
Hantavirus pulmonary syndrome. Atypical lymphocytes and premature white blood cells
may be present on admission, but leukocytosis alone is not significant enough to warrant
admission.
A 39-year-old man is diagnosed with pulmonary alveolar proteinosis following
bronchoscopy with lavage. His serum GM-CSF levels appear normal, but his anti-GM-CSF
IgG antibody levels are markedly elevated. What is the treatment of choice?
1. Systemic corticosteroids
2. Inhaled GM-CSF
3. Plasmapheresis
4. Whole lung saline lavage
Answer: 4 - Whole lung saline lavage
Explanations: The treatment of choice in pulmonary alveolar proteinosis is whole lung
lavage, which has been shown to improve mortality, symptoms, and respiratory function in
retrospective analyses. Although autoimmunity is the likely cause of his pulmonary
alveolar proteinosis, systemic corticosteroids have been found to worsen outcomes due to
risk of infections without improvement in disease. Autoimmune pulmonary alveolar
proteinosis represents about 90% of all pulmonary alveolar proteinosis cases, while
secondary and congenital pulmonary alveolar proteinosis represent 5% together. Whole
lung lavage may be curative in autoimmune pulmonary alveolar proteinosis after repetitive
treatments, whereas it is not curative in secondary or congenital pulmonary alveolar
proteinosis.
A 56-year-old male with a history of smoking and metastatic melanoma comes to the clinic
for shortness of breath. The patient started treatment with ipilimumab and nivolumab for
melanoma two months ago and has received two cycles. He reports exertional dyspnea for
3 to 4 days with a non-productive cough. He denies any preceding fever. The oxygen
saturation on room air was 93%. On examination, he is not in respiratory distress, and
bilateral crackles noted in lung bases. You obtain a stat CT chest without contrast that
shows bilateral ground-glass opacities without focal consolidation. How should this
patient be managed?
1. Admit the patient to the hospital for atypical pneumonia
2. Start outpatient broad-spectrum antibiotics.
3. Start prednisone 1 mg/kg
4. Refer to pulmonary for outpatient bronchoscopy
Answer: 3 - Start prednisone 1 mg/kg
Explanations: This patient likely has immune-mediated pneumonitis based on the facts
that he is afebrile, CT scan is showing ground glass opacity without an infiltrate, and he got
combination immunotherapy recently. Immune-mediated pneumonitis is seen in about 3%
of patients treated with single-agent anti-PD 1 agent such as nivolumab and about 6% of
patients treated with a combination of nivolumab with ipilimumab. Prednisone 1mg/kg
body weight is the initial drug of choice. Once clinical improvement is seen, prednisone
can be tapered. Immuno-suppressive agents such as mycophenolate have been shown to
have benefit in refractory cases. Starting broad-spectrum antibiotics is not warranted at
this point since the workup is not suggestive of an infectious etiology. Routine
bronchoscopy is not needed for diagnosing immune-mediated pneumonitis and can be
used when the etiology is uncertain or if the patient does not improve with
corticosteroids. Admitting this patient is not indicated at this point since he appears to be
oxygenating well.
A patient with cirrhosis of liver presents with anginal chest pain. A stress test is positive.
The patient undergoes cardiac catheterization, and a selective coronary angiogram is
negative for coronary artery disease. But the dyes injected into the right and left coronary
ostia filled the ventricles. Which of the following mechanisms is most likely responsible
for the patient's symptoms?
1. Steal phenomenon
2. Increased oxygen demand due to chamber hypertrophy
3. Increased blood flow to ventricles
4. Due to hypoalbuminemia
• 79

You might also like